You are on page 1of 166

Philip Kelly

A3§ rons
Book 1
Cardiology
Respiratory- Gastroenterology
Medicine Dermatology
Psychology
RenalMedicine

PASTEST
Dedicated to your Success
Contributors vii
Acknowledgements viii
Introduction ix

Haematology 1
Questions 3

Neurology 45
Questions 47

Ophthalmology 71
Questions 73

Rheumatology 79
Questions 81

Answers with teaching explanations 109


Haematology 11 1
Neurology 131
Ophthalmology 148
Rheumatology 152

Index 169

v
Haematology
Claire ) Hemmaway MRCP MRCPath
Specialist Registrar in Paediatric Haematology, Hammersmith Hospital,
London.

Neurology
David L H Bennett MB PhD MRCP
Specialist Registrar in Neurology , Department of Neurology ,
King's College Hospital, London.

Ophthalmology
Jasmin K Singh MBBS MRCOphth
Senior House Officer in Ophthalmology ,The Western Eye Hospital,
London.
Amir Hamid BMedSci BMBS MRCOphth
Senior House Officer in Ophthalmology , Manchester Royal Eye Hospital,
Manchester.

Rheumatology
William C Dixon MRCP
Clinical Research Fellow, arc Epidemiology Unit and Specialist Registrar
in Rheumatology, University of Manchester, Manchester.

vii
I would like to express my gratitude to the team at PasTest particularly
Cathy Dickens for her unswerving support and tolerance during the
preparation of this series. Many patients have been gracious enough to
contribute to our ongoing education by allowing their images to be used in
these volumes. The series would have been impossible without the help of
the following: Mrs Sue Hemmaway for preparation of the haematology
section; Dr Philip Beer and David Roper, Hammersmith Hospital, for help
with blood films; Dr Mark Layton, Consultant haematologist,
Hammersmith Hospital; Mr Riordan-Eva, Consultant ophthalmologist,
Kings College IHospital; Medical Photography, Radiology and Medicine at
King George Hospital, llford; Medical Illustration at Barts and The London
School of Medicine and Dentistry; Radiology and The Department of
Diabetes and Metabolism at Barts and The London NHS Trust.

Philip Kelly

viii
The MRCP (UK) Part 2 written examination consists of two 3 hour papers,
each with up to 1 00 multiple choice questions; they are either I from 5
(best of 5) or 'n' from many, where 2 answers are chosen from 1 0. Each
question will have a clinical scenario and might contain investigations to
interpret; many might also contain an image. There is a pass mark agreed
by the examiners but a candidate's performance is also assessed in relation
to other candidates.

This 3 book series provides practice questions with extensive explanations


to aid candidates preparing for the examination. The authors arc all
clinicians writing sections in their chosen fields and as such have been
chosen for their clear understanding of the required knowledge base for
this important exam. The breadth of knowledge for this exam is vast and
they have attempted to cover the 'syllabus' as completely as possible.
Great care has been taken to explain areas that cause difficulty as
thoroughly as possible. No apology is made where the format of the
questions differs slightly from the exam. These books are not merely
practice papers but educational aids and where a topic can be best
explained by diversion from the strict format of the exam, for the sake of
understanding, this has been done.
This book covers haematology , neurology , ophthalmology and
rheumatology and is best taken - in concert with its colleagues within the
series - as a supplement to a thorough clinical grounding, the general
medical texts and the core clinical journals.
Any comments or suggestions on this book or the series will be gratefully
received.

IX
HAEMATOLOGY - QUESTIONS

Case 1
A 38-year-old Nigerian lady was referred to the Gastroenterology Clinic
for investigation of hyperbilirubinaemia. Her past medical history
includeda cholecystectomy for gallstone disease more than 10 years ago.
On examination she was jaundiced and pale. There was no
lymphadenopathy. A splenic tip was palpable.
Her blood results were as follows:
Hb 8.8 g/dL
Reticulocytes 4% (absolute count 300 X o r—
WCC 8.9 x 107L
Platelets 156 x 107L
MCV 96 fL
MCH 30 pg
MCHC 37 g/dL
Hb electrophoresis HbF0.9% HbA2 2.8% HbA 96%
U&Es Normal
Bilirubin 56 (imol/L
ALP 100 U/L
AST 30 U/L
ALT 30 U/L
vCT 32 U/L

1 What are the first three investigations you must do?


ÿ A Blood film
ÿ B Haptoglobins
ÿ C Split bilirubin
ÿ D Osmotic fragility
ÿ E Heat stability test
ÿ F Liver/gallbladder ultrasound
ÿ G Direct Coombs' test
ÿ H Malaria film
ÿ I Glucose-6-phosphate dehydrogenase (G6PD) levels

3
HAEMATOLOGY- QUESTIONS

2 Which three diagnoses are in the differential diagnosis?


ÿ A A membrane disorder, eg hereditary spherocytosis (HS)
ÿ B G6PD deficiency
CD C Autoimmune haemolytic anaemia (AIHA)
CD D A haemoglobinopathy
CD E Haemolytic uraemic syndrome (HUS)
ÿ F Disseminated intravascular coagulation (DIC)
CD G Thrombotic thrombocytopenic purpura (TTP)
CD H Malaria
CD I Paroxysmal nocturnal haemoglobinuria (PNH)
ÿ J Gilbert's syndrome

The film is shown below:

'

oV O—
dfi OaO
, , ft '
ÿ

,
•5 « * >;
v ••*i e
1 c« °o°o
s c ••

3 What is the diagnosis?
ÿ A A membrane disorder, eg HS CD F DIC
ÿ B G6PD deficiency ÿ G TTP
ÿ C AIHA ÿ H Malaria
ÿ D A haemoglobinopathy ÿ 1 PNH
ÿ E HUS ÿ J Gilbert's syndrome

4
HAEMATOLOCY - QUESTIONS

Case 2

A / 6-year-old Nigerian girl presented to A&E with weakness down the left
side of her body. She hadbeen in a lesson at school when the weakness
developedsuddenly. She was on no medications andhad no past medical
history apart from mild intermittent joint and bone pains. Her mother had
been diagnosed with SLE 5 years earlier. Her father was well and worked
in the (lily as an accountant. She had one sister who was well. She had
been born prematurely at 25 weeks.
On examination she was jaundiced and had 4/5 weakness affecting her
left upper and left lower limbs. Her face was spared. A systolic murmur
was heard at the left sternal edge.

On further questioning, she maintained that her sclera were always


yellow.

Her blood test results were as follows:


Hb 8.0 g/dL
MCV 90 fL
WCC 16x10'VL
Platelets 550x107L
Reticulocytes 5% (absolute count 200 x 107L)
OCT Negative
U&Es Normal
Bilirubin 35 pmol/L
ALP 90 U/L
AST 25 U/L
ALT 30 U/L
LDH 1500 U/L
PT 11s
APTT 30 s
TT 18 s
Fibrinogen 2.5 g/L

1 Which investigation would you do first?


ÿ A Thrombophilia screen D F Lumbar puncture
ÿ Magnetic resonance
B O G Hb electrophoresis
angiography (MRA) U H Carotid Doppler
ÿ C Lupus anticoagulant I I Echocardiogram
ÿ D Autoimmune profile LI ) Homocysteine levels
C E Anti cardiolipin antibodies

5
HAEMATOLOGY- QUESTIONS

2 How would you manage this patient?

CD A Aspirin
ÿ B Thrombolytic therapy
ÿ C Blood transfusion
ÿ D Exchange blood transfusion
ÿ E Warfarin
ÿ F Heparin
ÿ G Steroids
ÿ H Carotid endarterectomy
ÿ I Closure of ventriculo-septal defect

6
HAEMATOLOGY - QUESTIONS

Case 3

A 25-year-old Asian lady presented to her CP again with increasing


tiredness,lethargy and easy bruising.
Her blood results were as follows:
Hb 8.2 g/dL
MCV 69 fL
MCH 21 pg
RCC 3.8 x 10'2/L
RDW 24
Platelets 500 x 10q/L
WCC 10.9x109/L
Neutrophils 7.7 x 109/L

The film is shown below:

O»o« o* °Vtf
8oo°
W
•o 9° 9* 9ÿ°o °{

oofooto"
Son
, ° ®°0
C O O*
.*> SPÿ6?o°T
1 What is the diagnosis?
Q A a-Thalassaemia trait
ÿ B |3-Thalassaemia trait
ÿ C Acute blood loss
ÿ D Iron deficiency anaemia
ÿ E Acute myeloid leukaemia (AML)

7
HAEMATOLOGY - QUESTIONS

2 What three features shown in the blood film suggest the diagnosis?
ÿ A Acanthocytes
ÿ B Pencil cells
[ I C A dysplastic neulrophil
ÿ D Target cells
ÿ E Macrocytes
ÿ F Hypochromia
ÿ G A hyperlobulated neutrophil
ÿ H Hyperchromia
CD I Microcytosis
ÿ | Howel l-lolly bodies

8
HAEMATOLOGY - QUESTIONS

Case 4

A 27-year-old Afro-Caribbean man presented to A&E with priapism.


Otherwise he felt well in himself. The only other finding on examination
was a left upper quadrant mass, which extended 7 cm below the
costophrenic angle.
Full blood count results were as follows:
Hb 11 g/dL
WCC 700 x 10'VL
Platelets 1400 x 10'VL

The film is shown below:

1 What is the diagnosis?


ÿ A Sickle cell disease (SCD)
LJ B Acute myeloid leukaemia (AML)
ÿ C Chronic myeloid leukaemia (CML)
D D Acute lymphoblastic leukaemia (ALL)
D E Chronic lymphocytic leukaemia (CLL)
ÿ F High-grade non-Hodgkin's lymphoma (NHL)
CD C Follicular lymphoma
ÿ H Sildenafil overdose

9
HAEMATOLOCY QUESTIONS

2 Which cytogenetic abnormality is associated with this disorder?


ÿ A t(4;1 1)
ÿ B +13
ÿ C +19
ÿ D t(9;22)
ÿ F t(1 4; 18)
ÿ F 1(8;14)
ÿ G t(8;21)
ÿ H t(9;1 1)

10
HAEMATOLOCY - QUESTIONS

Case 5

/A 29-year-old Afro-Caribbean man with CML receives a sibling bone


marrow transplant.
Seventeen days after the transplant his counts are:
Hb 8.5 g/dL
WCC 0.1 x 10'VL
Neutrophils O.OxlO'VL
Platelets 15x10'VL
CRP 245 mg/L

He develops fevers of 40 °C, pleuritic chest pain and haemoptysis. He


denies shortness of breath at rest or on exertion. His oxygen saturations are
97% on air.

The CXR and CT scan are shown below and on the next page respectively:

11
HAEMATOLOGY- QUESTIONS

1 What infection would you be most suspicious of with this history and
radiological appearance?
ÿ A Pneumocystis pneumonia (PCP)
ÿ B Fungal chest infection
ÿ C Cytomegalovirus (CMV) pneumonitis
ÿ D Respiratory syncytial (RSV) pneumonitis
ÿ E Ftospital-acquired bronchopneumonia
0 F Tuberculosis
ÿ C Staphylococcus aureus abscess

12
HAFMATOLOGY - QUESTIONS

Case 6

A 65-year-old Moroccan diplomatpresented to hospital with a 3-month


history of fatigue. He was a heavy drinker but did not smoke. He had
travelled widely as part of his job - throughout Africa, Asia, Europe and
America. He had a past history of rheumatoid arthritis, treated in the past
with steroids, but the disease was currently quiescent. He hadhad a deep
venous thrombosis (DVT) 6 years earlier for which he hadbeen treated
with warfarin for 6 months.
On examination he had a low-grade fever, he was pale, jaundiced and had
hepatosplenomegaly with the liver being 5 cm below the costal margin
and the spleen 17 cm below the costal margin. There was no peripheral
lymphadenopathy and no evidence of an active arthropathy.

His blood results are shown below:


lib 6.7 g/dL
WCC 3.0 x 107L
Neutrophils 1.9x107L
Lymphocytes 0.7 x 107L
Platelets 83 x 107L
Reticulocytes 3.85% (absolute count 250 x 1 07L)
Film Polychromasia
Target cells
Spherocytes
Thrombocytopenia
ESR 132 mm/h
B12 and folate Normal
Serum iron 12 umol/L
TIBC 40 umol/L
Ferritin 300 pg/L
Transferrin saturation 10%
DCT positive IgG 5+, C3D 4+
1 laptoglobins Reduced
U&Es Normal
Bilirubin 37 pmol/L (conjugated 8 pmol/L)
AST 80 U/L
AIT 60 U/L
ALP 100 U/L
yGT 63 U/L
LDH 530 U/L
Protein electrophoresis IgM kappa paraprotein 2 g/L
Serum cryoprotein Not detected

13
HAEMATOLOGY - QUESTIONS

1 What is the main cause of the anaemia?


ÿ A Iron deficiency
ÿ B Acute blood loss
CD C Anaemia of chronic disease
CD D Myelodysplasia
CD E Non-Hodgkin's lymphoma
CD F Haemolysis
' i G Hypersplenism
CD H None of the above

2 Choose three possible differential diagnoses from the following list:


I J A Lymphoproliterative disorder
C B Chronic liver disease
CD C Autoimmune haemolytic anaemia
. 1 D Acute myeloid leukaemia
CD E Multiple myeloma
i ) F Chronic myeloid leukaemia
CD G Myelofibrosis
CD H Acute hepatitis

14
HAEMATOLOGY - QUESTIONS

Case 7
A 55-year-old man presented to his CP feeling tired, weak andgenerally
unwell. He gave a history of increasing shortness of breath over the last
few weeks anda non-productive cough. His past medical history included
hypertension, stable angina, diabetes mellitus and rheumatoidarthritis.
On examination, he was pale and had pitting ankle oedema. His BP was
170/100 mrnHg. Otherwise, examination was unremarkable.
His bloods counts were as follows:
Hb 6.3 g/dL ESR 75 mm/h
MCV 89 fL U&Es Normal
Platelets 155 x 1 0'VL LFTs Normal
WCC 10.2 x 107L TSH 0.2 mU/L
Neutrophils 5.6 x 10"/L t4 9 pmol/L
Reticulocytes 0% (absolute count 0 x 1 0'VL)

His CXR is shown below:

15
HAEMATOLOCY - QUESTIONS

1 What are the differential diagnoses, based on the CXR alone?


(five answers)

o A Hodgkin's disease
o B Mediastinal non-Hodgkin's lymphoma
o C Lung carcinoma
0 D Dissecting thoracic aortic aneurysm
0 E Teratoma
0 F Sarcoidosis
0 G Tuberculosis
ÿ H Pneumonia
0 I Thymoma
o ) Multiple myeloma
o K Thyroid tumour
o L Syphilitic aortitis

While being investigated for her abnormal CXR she requires a blood
transfusion every 3-4 weeks. Her anaemia is investigated:
Bi2 160 ng/L
Folate 7 ng/L
Ferritin 200 ng/L
Film Normocytic anaemia with a complete absence of
polychromasia

2 What is the cause of the anaemia?


ÿ A Myelodysplasia
0 B Parvovirus infection
0 C Carcinoma of the lung with bone marrow involvement
0 D Red cell aplasia
0 E Anaemia of chronic disease
0 F Mixed haematinic deficiency
0 G Acute blood loss
On further questioning this lady also complains of diplopia while reading.

3 What additional investigation could you perform?


0 A Thyroid autoantibodies
0 B LP and cytology
0 C Parvovirus titres
0 D CT orbit
0 E Bronchoscopy
O F Tensilon0 test

16
HAEMATOLOGY - QUESTION'S

Case 8

A 60-year-old Nigerian gentleman presents in the UK for a second


opinion. Three years before he hadbeen toldhe was anaemic. He had
splenomegaly to below the umbilicus and a liver edge. He had a bone
marrow in Nigeria which was inconclusive. His past medical history
includes hypertension. He has travelled extensively throughout Africa and
has had recurrent attacks of malaria, treated with chloroquine and, most
recently, Paludrind*.
I le is pale and has splenomegaly as described above. He has no stigmata
of liver disease. On auscultation of his heart sounds he has a systolic
murmur, loudest at the apex. There is no palpable lymphadenopathy.

I lis investigation results are shown below:


Hb 5.9 g/dL
MCV 85 fL
MO 1 27.4 pg
WCC 6.6 x 1071
Neutrophils 4.8 x 1071
1 ymphocytes 1.3 x I07L
Platelets 120 x 107L
Reticulocytes 3.3% (absolute count 83 x 107L)
C6PD 205 U/gHb (raised)
Folate 10 pg/L
B,2 300 ng/L
Ferritin 706 p.g/L
Serum iron 10 u.mol/L
TIBC 56 u.mol/L
Malaria film Negative
U&Es Normal
Bilirubin 26 itmol/l
ALP 1 00 U/L
ALT 30 U/L
AST 30 U/L
1 Dl 1 807 U/L
Uric acid 0.45 mmol/L
Albumin 40 g/L
OCT Negative
Hep A/B/C serology Negative (IgG and IgM)
US abdomen Spleen 2 Icm

17
HAEMATOLOGY- QUESTIONS

1 What are the three most likely diagnoses?


ÿ A C.ML
( ) B Amyloidosis
ÿ C Chronic liver disease
0 D Gauchcr's disease
ÿ E Leishmaniasis
0 E Myelodysplasia
0 G Myelofibrosis
0 1 1 Tropical splenomegaly
0 I Felly's syndrome
0 ) Sickle cell disease
ÿ l< AML
( ) I. Autoimmune haemolytic anaemia (AIHA)
O M CLL

2 Which three diagnostic tests would you do to differentiate between your


three diagnoses?
0 A Bone marrow
0 B Rectal biopsy and stain with Congo red
0 C Splenic biopsy
LJ D Leishmania serology
C ) E Rheumatoid factor
0 F Malarial antibody litres
0 G Leucocyte glucocerebrosidase activity
i ) H CT chest/abdomen/pelvis

18
HAEMATOLOGY - QUESTIONS

3 The blood film is shown below: what two features are shown?

ÿ A Tear-drop poikilocytes
CD B Basophilic stippling
CD C Hypochromia
ÿ D Howell-Jolly bodies
CD E Heinz bodies
CD F Polychromasia
CD G Microcytosis
CD H Circulating blasts
ÿ I Leucoerythroblastic blood film

19
HAEMATOLOGY - QUESTIONS

Case 9
A 5 1-year-old Caucasian gentleman presented with a flu-like illness anda
symmetricalpolyarthropathy affecting his knees, ankles, shoulders,
elbows and wrists. There was obvious swelling of his knees andankles and
a papular rash over his arms. A full examination was otherwise
unremarkable. He was initially treated with antibiotics by his GPhut failed
to improve. He then presented to A&E with the same symptoms and was
admitted for investigation andcommenced on anti-inflammatories. X-rays
were carried out of all the involvedjoints, which showed no bony
erosions. There was some improvement andhe was discharged after a few
days.
6 weeks later he was readmitted with shortness of breath, a cough and a
persist arthropathy. The rash had resolved. He remained apyrexial.

The CXR is shown below:

20
HEMATOLOGY QUESTIONS

His blood results are shown below:


Hb 15.3 g/dL
WCC 7.0 x 10'VL
Platelets 326 x I07L
MCV 92 II
LSK 34 mm/h
U&Es Normal
ALP 90 U/L
AST 40 U/L
ALL 42 U/L
Bilirubin 13 umol/L
Corrected Ca~ 2.2 mmol/L
Phosphate 0.9 mmol/L
CRP 153 mg/L

Blood gases:
PH 7.4
Pco2 5.48 kPa
Po, 9.89 kPa
(>> saturations 95%

I le went on to have a bronchoscopy and lungbiopsy which showed


non-caseating granulomas and a lymphocytosis (41%) on
bronchoalveolar lavage (BAL).

1 What is the most likely diagnosis?


O A Tuberculosis
CD B Teratoma
CD C Parvovirus infection
ÿ D Brucellosis
D E Rheumatoid arthritis
fl F Sarcoidosis
I I C Hodgkin's disease
( J H Lung carcinoma
i 1 I Cat scratch disease
I J Non-F lodgkin's lymphoma
t_ K Polyarteritis nodosa

21
HAEMATOI.OGY - QUESTIONS

He is treated for his underlying condition successfully. He continued to be


followed up regularly. At one of these routine follow-ups he has the
following full blood count:
Hb 18.2 g/dL
KCC 6.4 x 10,2/L
Hct 0.52
MCV 80 fL
WCC 11x10'7L
Neutrophils 7x 107L
Platelets 400xl0"/L
SpQ2 91% on air

2 What is the diagnosis?


0 A Essential thrombocytosis
ÿ B Primary polycythaemia rubra vera
ÿ C Secondary polycythaemia
ÿ D Reactive thrombocytosis
ÿ E Iron deficiency
ÿ F B12 deficiency
ÿ G Neutrophilia secondary to steroids
ÿ H Dehydration
0 I Chronic myeloid leukaemia

22
IIAEMATOLOGY - QUESTIONS

Case 10

An 80-year-old Indiangentleman, who has been in this country for 30


years but who frequently goes back to India (last visit more than Iyear
previously), presented to his CP non-specifically unwelland had a large
bruise over his hip following a fall. His past medical history included
hypertension, angina and hypercholesterolaemia.
On examination he was apyrexial. His BP was 180/70 mrnHg. There were
numerous bruises ( <2 cm) over his lower limbs and a 10 cm x 10cm
bruise over his left hip.

I lis blood results are shown below:


Hb 10.2 g/dL
MCV 86 fL
WGC I x 107I
Neutrophils 0.02 x 1071.
Lymphocytes 0.2 x 1071.
Platelets 14x1071
Reticulocytes 0.68% (absolute count 21.9 x I()7L)
G6PD activity Normal
Sodium 1 54 mmol/L
Potassium 3.9 mmol/L
Urea 9.5 mmol/L
Creatinine 133 umol/L
Bilirubin 37 pmol/L
ALT 39 U/L
ALP 56 U/L
Bone profile Normal
CRP 9.5 mg/l

1 Which three diagnoses are at the top of the differential diagnosis?


[ i A Autoimmune haemolytic anaemia
ÿ B Immune thrombocytopenia
ÿ C Iron deficiency anaemia
LJ Non-Hodgkin's lymphoma
D
O Myelodysplasia
E
Q E Chronic lymphocytic leukaemia
D C Acute myeloid leukaemia
D II Hairy cell leukaemia
ÿ I I IIV/AIDS

23
HAEMATOLOGY- QUESTIONS

2 The bone marrow is shown below: two diagnoses can be made. What are
they?

ÿ A Acute myeloid leukaemia ÿ F Falciparum malaria


ÿ B Hairy cell leukaemia ÿ C Leishmaniasis
ÿ C Chronic lymphocytic leukaemia ÿ H Gaucher's disease
ÿ D Non-Hodgkin's lymphoma ÿ 1 Ovale malaria
ÿ E Tuberculosis ÿ 1 Haemophagocytosis

24
HAEMATOLOGY QUESTIONS

Case 11

A 26-year-oldgentleman with sickle cell disease who had recently


returned from The Gambia, where he was born, presented to A&E. He
complained of abdominalandback pain. Ilis past medical history
included recurrent painful crises and two episodes of chest crisis which
requiredITUadmission. He hada cholecystectomy 5 years ago because of
recurrent cholecystitis.

He was on hydroxycarbamide (hydroxyurea), 1g daily, which had


reduced the frequency of his admissions for painful crises dramatically.

On examination he was pale, pyrexial (38.5 !'C), significantly jaundiced


and had generalised abdominal tenderness. Chest auscultation was clear.
His oxygen saturations were 99% on air.

I lis blood results are shown below:


Hb 6.0 g/dL
WCC 17x10''/L
Platelets 500 x 107L
Neutrophils 15x107L
MCV 112 fL
U&Es Normal
Bilirubin 78 jimol/L
ALP 1 10 U/l
AST 26 U/L
AIT 32 IJ/L
Bone profile Normal

1 What are the first three investigations you would do?


C A Group and save
ÿ B Haemoglobin S %
ÿ C Chest X-ray
ÿ D Ultrasound (US) of the abdomen
ÿ E Malaria film
ÿ F G6PD levels
ÿ G Blood cultures
ÿ H Urine haemosiderin
ÿ I B u and folate

25
HAEMATOLOGY - QUESTIONS

2 The blood film is shown below. How would you treat this complication?
(one answer)

V•* *
A.
3
*
rVv
# ' o° V

ÿ A Broad-spectrum antibiotics
-ST
CD B Oxygen
CD C Red cell transfusion
CD D Analgesia
CD E Intravenous fluid
ÿ F Incentive spirometry
CD G Physiotherapy
CD H None of the above
CD I All of the above

3 What investigation should be done prior to commencing therapy?


ÿ A Chest X-ray
ÿ B Blood cultures
ÿ C Haemoglobin S %
ÿ D Ultrasound of the abdomen
ÿ E Reticulocyte count
CD F G6PD levels
CD G Group and save

26
I IAFMATOLOGY - QUESTIONS

Case 12

/I 45-year-old Caribbean gentleman was referred to Haematology


Outpatients with widespread small-volume lymphadenopathy, night
sweats, weight inss and diarrhoea. He admitted to heavy drinking (in
excess of 50 units a week) and was a smoker (50 a day). He had had
multiplesexual partners in the last 20 years. He had been born in Jamaica,
but had lived in the UK for 2 years. He returned to Jamaica frequently.
On examination he had a low-grade pyrexia, widespread small-volume
lymphadenopathy, hepatosplenomegaly (liver 2 cm below the costal
margin and a splenic tip).

His investigation results were as follows:


Hb 1 1 .6 g/dL
wee 10.4 x 1 0'VL
Neutrophils 4 x 10'YL
Lymphocytes 6 x 1 07L
Platelets 140 x 1 0'YL
MCV 94 fl
U&Es Normal
Bilirubin 1 8 pmol/L
Albumin 28 mmol/L
ALT 50 U/L
AST 55 U/L
ALP 140 U/L
ID1 1 900 U/L
Calcium 2.7 mmol/L
Phosphate 0.7 mmol/L
CXR Normal

1 What is the most likely diagnosis?


ÿ Sarcoidosis
A
ÿ Tuberculosis
B
ÿ I luman immunodeficiency virus (HIV)
C
ÿ Adult T-cell leukaemia/lymphoma (ATLL) secondary to HTI.V-
D
(human T-cell leukaemia virus infection)
D E Iiodgkin's disease
ÿ F Sezary syndrome
ÿ c; I lyperparathy roidis m
ÿ H Metastatic carcinoma of the lung

27
HAEMATOLOCY - QUESTIONS

2 He has diarrhoea: what is the cause?

0 A HIV enteropathy
ÿ B Cytomegalovirus (CMV) enteritis
0 C Cryptosporidium infection
0 L) Clostridium difficile infection
0 E Tuberculous enteritis
0 F Strongyloides infection

28
HAEMATOLOGY QUESTIONS

Case 13

A 54-year-old lady presented to her CP with increased bruising and


tiredness. She had been diagnosed with chronic myeloid leukaemia 19
years before. She was initially treated with interferon and then went on to
have an autologous bone marrow transplant. She hadbeen completely
well for 19 years. Cytogenetic analysis hadalways shown 0% Philadelphia
chromosomes since her transplant.
On examination she had numerous bruises, but there was nothing else to
find.

Her blood results showed the following:

Hb 1 0.2 g/dL U&Es Normal


WCC 0.6 x 10"/L Bilirubin 22 pmol/L
Platelets 4x10"/L ALT 40 U/L
Neutrophils 0.1 x 10''/L AST 35 U/L
MCV 89 fL ALP 90 U/L

The film showed thrombocytopenia, leucopenia, a normocytic anaemia


and an occasional circulating myeloid precursor.

The bone marrow is shown below:

29
HAEMATOLOCY QUESTIONS

Cytogenetic analysis showed the presence of Philadelphia chromosomes.

1 What is the diagnosis?


0 A Immune thrombocytopenia (UP)
ÿ

) B Myelodysplasia
O C Chronic myeloid leukaemia (CMC) in chronic phase
J D Chronic myeloid leukaemia (CML) in blast crisis
O E Acute myeloid leukaemia (AML)
_l F Acute lymphoblastic leukaemia (ALL)

The cells shown in the bone marrow have the following markers on their
surface:
CD19 positive CD7 negative
TDT positive CD13 negative
CD34 positive CD33 negative
CD10 positive CD2 negative
CD79a positive CD3 negative

2 Which lineage is the current problem in?


0 A B lymphoid
O B Myeloid
0 C T lymphoid
0 D None of the above

30
HAEMATOLOGY - QUESTIONS

Case 14

A 32-year-oldlady presentedto A&E with bleedingper vaginam. She is 20


weeks pregnant. On ultrasound there is intrauterine growth retardation
(IUCR). She hada DVT aged 30 following a road traffic accident when she
fractured her left femur.
I ler blood results were as follows:

lib 1 0.5 g/dL


VVCC 11x107L
Platelets 51 x 107L
MCV 78 fL
PT 15 s
A Pit 58 s
50:50 mix 58 s
TT 12s
Fibrinogen 2.5 g/dL

1 Which three investigations would you request next to investigate the


abnormal blood results?
ÿ A Factor VIII levels
I . I? Lupus anticoagulant and anti cardiolipin antibodies
Ii C Buand folate
[ I D Bone marrow
ÿ E Urate
( ) F Blood pressure
ÿ (i Blood film
ÿ H Dipstix urinalysis
ÿ I von Wi Ilebrand factor level
f I | Haemolysis screen
ÿ K All of the above
ÿ L None of the above

31
HAFMATOLOGY - QULSTIONS

2 What other history would you like to know? (one answer)


ÿ A Past obstetric history
ÿ B Family history of thrombotic episodes
O C Bleeding history
ÿ D Family history of early pregnancy loss
ÿ E Family history of thrombocytopenia
ÿ F Weight loss
ÿ G Night sweats
ÿ Fl Warfarin-induced skin necrosis

3 What diagnosis would you consider?


ÿ A Acquired haemophilia A
O B von Willebrand's disease
D C Immune thrombocytopenia
ÿ D Antiphospholipid syndrome
ÿ E Thrombotic thrombocytopenic purpura

32
HALMATOLOGY - QUESTIONS

Case 15

/A 25-year-old lady who is 34 weeks pregnant (para 2) presented to A&E


with a 4-day history of jaundice,easy bruising and a severe headache.
There hadbeen no prior problems in the pregnancy.
On examination she was jaundiced. Her BP was 1 10/75 mmHg. Her
temperature was 37 °C. She had several small bruises over her abdomen
and legs. Urinanalysis showed protein +++• and blood +++.

Her blood results were as follows:


Hb 8.6 g/dl
WCC 5.6 x 10'VL
Neutrophils 4.0 x 107L
Platelets 7 x 109/L
MCV 90 fL
Reticulocytes 4% (absolute count 200 x 1 07L)
U&Es Normal
Bilirubin 155 umol/L
AST 25 U/L
ALT 25 U/L
ALP 95 U/L
LDH 5000 U/L
Clotting and fibrinogen Norma

33
HAEMATOLOGY- QUESTIONS

The film is shown below:

*
!•

» ••••
O
••
o •• \

•• • • ©
>•• *
1 What is the most likely diagnosis?
ÿ Thrombotic thrombocytopenic purpura (TTP)
A
ÿ Haemolytic uraemic syndrome (HUS)
B
ÿ C HELLP syndrome
CD D Pre-eclampsia
ÿ E Immune thrombocytopenia

2 What is the correct, evidence-based management?


ÿ A Folic acid
CD B Plasma exchange
CD C Dialysis
ÿ D Steroids
ÿ E Liver biopsy
ÿ F Intravenous immunoglobulin (IVIG)
CD G Bed rest
CD H Emergency Caesarean section
I None of the above

34
HAEMATOLOCY QUESTIONS

Case 16

A 76-year-oldPolishgentleman presented to A&f with a 3 -week history of


lethargy, shortness of breath, confusion and being 'off his legs'.
On examination he had a chaotic pulse ol 1 40 bpm. I lis BP was 1 30/85
mmHg and his SaO> 99% on air. He was jaundiced and had bibasal
inspiratory crepitations on auscultation of his chest. It was impossible to
do a full neurological examination on him because he spoke no English
and was contused.

His bloods results are shown below:


Hb 5.9 g/dL
wee 4.0 x 10'VL
Neutrophils 2.8 x 107L
MCV 129 t'L
Platelets 90 x 1 0"/L
Reticulocytes 0.5% (absolute count 25 x lO'VL)
DCT Negative
Clotting and fibrinogen Normal
U&Es Normal
Bilirubin 50 pmol/l
AST 3 .5 U/L
ALT 3.5 U/L
ALP 100 U/L
LDI I 2500 U/L
T-,4 9 pmol/L
TSH 0.2 mU/L

35
HAEMATOLOGY - QUESTIONS

The film is shown below:

1 What are the two differential diagnoses?


ÿ A Bi» or folate deficiency
D B Thrombotic thrombocytopenic purpura
D C Immune thrombocytopenic purpura
D D High-grade non-Hodgkin's lymphoma
ÿ E Hypothyroidism
ÿ F Hepatitis
ÿ G Myelodysplasia
ÿ H Autoimmune haemolytic anaemia

2 What would your next investigation be?


O A B12 and folate measurements
CD B Bone marrow
CD C Bone marrow chromosome analysis
ÿ D TSH test
ÿ E CT chest/abdomen/pelvis
ÿ F Haptoglobins

36
IIAEMATO LOG Y - QUFSTIONS

3 Which (wo are (he correct management?


ÿ A B 12 replacement
ÿ B Blood transfusion
ÿ C Chemotherapy
D D Digoxin
ÿ E Plasma exchange
ÿ F Thyroxine

37
HAEMATOLOGY- QUESTIONS

Case 17

A 60-year-old man is admitted via his CP to your care with streptococcal


pneumonia and is treated appropriately. He is a smoker, drinks a bottle of
vodka per week and works in a fish market during the week. 10 days after
his admission his haemoglobin drops to 9g/dL from 12 g/dL on admission
andhe becomes jaundiced. He hadbeen started on ibuprofen for
generalised aches andpains.
The next day the blood results were as follows:
Hb 6.4 g/dL
MCV 99 fL
WCC 14.6 x 10''/L
Neutrophils 1 1 x 10'VL
Platelets 140 x 107L
Reticulocytes 6% (absolute count 300 x 107L)

The film is shown below:

bo? 02

*
Oa 09
o**e oy*
<? 3,f«. *
38
HA EMATO LOG Y - QUESTIONS

1 Which two investigations would you do next?


L A B12 measurement
Q 13 Direct Coombs' test (DCT)
ÿ C LDI I
D D Ferritin
CD E Oesophagogastroduodenoscopy
CD F Folale
CD G Colonoscopy
CD H None of the above

2 What is the diagnosis?


ÿ Delayed transfusion reaction
A
CD Immune haemolytic anaemia
B
CD C Gastrointestinal bleed
CD D Iron deficiency
CD E Folate deficiency
CD F B 12 deficiency

3 What is implicated in the aetiology of the above diagnosis?


CD A A blood transfusion
CD B Ibuprolen
CD C Folate
CD D Peptic ulceration
CD E Antibiotic treatment

39
HAEMATOLOCY- QUESTIONS

Case 18

A 60-year-old gentleman presented to his CP with increasing tiredness,


shortness of breath and worsening of his angina over the last 7 0 days.
On examination he was mildly jaundiced and pale. He had no palpable
lymphadenopathy or organomegaly. Chest auscultation revealed no
abnormality. On auscultation of his heart he had a pansystolic murmur,
loudest at the apex.
His blood results were as follows:
Hb 5.1 g/dL
MCV 101 fL
WCC 140x10"/L
Platelets 200x10"/L
Reticulocytes 6.2% (absolute count 350 x 10'7L)
His blood film is shown below at low power and opposite at high power:

'v.-.:

40
HAEMATOLOGY - QUESTIONS

• f%
e«o*
o
• «- „ "
Oo 0 0
° • J0 _ °
e
ec
4;s •.••" t
ÿ A

)• w o O
o o • o
0 P ÿ c
1 Whal are the two diagnoses shown in the film?
ÿ A Hodgkin's disease
CD B Diffuse large B-cell lymphoma
ÿ C Chronic myeloid leukaemia (CML)
CD D Autoimmune haemolytic anaemia (AIHI)
CD E Iron deficiency
CD F Chronic lymphocytic leukaemia (CLL)
CD C Sezary syndrome
ÿ H B,2 deficiency
ÿ I Myelofibrosis
CD ) Acute lymphoblastic leukaemia (ALL)

The following are options to treat the two conditions:


1 Oral chemotherapy
2 B«
3 Ferrous sulphate
4 High-dose steroids
5 Intravenous chemotherapy
6 Radiotherapy
7 Splenectomy
8 Folic acid
9 Blood transfusion.

41
HAEMATOLOCY - QUESTIONS

2 Choose from the following combinations of treatment:


LJ A 7 and 8
B 1,4, 8 and 9
LJ C 5 and 8
ÿ D 8 and 9
ÿ E 4 and 8
O E 2 and and 4
CD G None of the above

42
IAEMATOLOGY - QUESTIONS

Case 19

A 33-year-oldman is referred In you by his CP with shortness of breathand


tiredness. Ile has hereditary spherocytosis. Eight days prior to the onset of
symptoms he had a mildfebrile illness, as didhis 4-year-old son.
On examination he is pale, jaundiced and a splenic tip is palpable.

Investigations reveal:
I lb 4.5 g/dF
MCV 90 fl
Platelets 1 60 x 1 07L
wee 1 0 x 1 0"/F
Lymphocytes 70%
Reticulocytes 0.1% (absolute count 5 x 10'VF)
Film Occasional atypical lymphocytes
Bilirubin 26 umol/L
U&Es and LFTs Normal

1 What is the most likely diagnosis?


CD A Folic: acid deficiency
CD B B i » deficiency
CD C Large gastrointestinal bleed
ÿ D Parvovirus-induced red cell aplasia
i E Hypersplenism
ÿ F Acute haemolysis
LJ G Epstein-Barr virus infection
CD 1 1 Cytomegalovirus infection

This patient had required a cholecystectomy at the age of 25.

2 What condition could exacerbate the propensity to gallstone formation?


ÿ A Gilbert's syndrome
LJ B B i2 deficiency
ÿ C Folate dependency
CD D Iron deficiency
LJ E Acute bleed
ÿ E High-fat diet

43
HAEMATOLOGY - QUESTIONS

Case 20

An Asian lady with recurrent iron deficiency has the following clotting
profile:
PT 12.5 s
APPT 44 s
TT 15 s (10-15 s)
Fibrinogen 5.2 g/L
Bleeding lime 1 I min
Factor VIII level 30 U/dL

1 What is the possible diagnosis, to explain her recurrent anaemia?


O A Vitamin K deficiency
0 B Haemophilia A
0 C von Willebrand's disease
0 D Warfarin overdose
0 E Amyloidosis
0 F Hyperfibrinogenaemia
0 G Hypofibrinogenaemia
0 H Ehlers- Danlos syndrome
0 I Systemic lupus erythematosus (SLE)
O | Haemophilia B
0 K Factor XI deficiency

44
NEUROLOGY - QUESTIONS

Case 1

A 28-year-old man presents to A&F complaining of difficulty swallowing,


dry mouth, altered speech anddouble vision over the last 5 days, and
today he had difficulty walking. He has a 10-year history of intravenous
heroin use.
On examination he is apyrexial, he has a bilateral ptosis, reduced
abduction of both eyes and his pupils are not reactive to light. He has
bilateral facial weakness, he is dysarthric and his cough is bovine.
Examination of his limbs shows that he is generally thin, Ihere are
multiple marks from injection sites, he has a proximal weakness in the
upper and lower limbs, his reflexes are reduced but present, plantar
reflexes are flexor, sensory examination is normal. His respiratory rate is
20/min; oxygen saturation is 96% on air; forced vital capacity is 900 mL.

Creatine kinase (CK) 1 90 U/l

1 What is the diagnosis?


ÿ A Guillain-Barre syndrome
CD B Myasthenia gravis
ÿ C Polio
ÿ D Wound botulism
CD E Drug-induced myopathy

2 Your next management step is to:

CD A Give botulinum antitoxin and penicillin


CD B Give pyridostigmine
CD C Give intravenous immunoglobulin
CD D Get urgent anaesthetic review for transfer to ITU
CD E Transfer to a neurology centre for plasma exchange

47
NEUROLOGY - QUESTIONS

Case 2

You arc referred a 70-year-old right-handed man in the Rapid-Access


Stroke Clinic. He describes an episode which occurred 2 months ago.
He was eating and developed acute-onset weakness and numbness of his
left arm. This lasted 5 minutes andthen completely resolved. Ile is
hypertensive andsmokes 10 cigarettes a day. His current medication is
aspirin 75 mg and ramipril 5 mg.
On examination his blood pressure is 130/86 mmHg, a carotid bruit is
heard on the right, neurological examination is normal.
He has had a number of investigations:
FBC Normal
Renal function Normal
Fasting glucose 3.9 mmol/1.
Fasting cholesterol 5.0 mmol/L
ECG Normal sinus rhythm
Echocardiogram Left ventricular hypertrophy
Carotid Doppler 55% stenosis of the right carotid artery

1 The appropriate management is to:


O A Increase the dose of aspirin
0 B Add dipyridamole to aspirin
0 C Replace aspirin with clopidogrel and simvastatin
O D Add dipyridamole and simvastatin
0 F Add dipyridamole and refer the patient for carotid endarterectomy

48
NEUROLOGY - QUESTIONS

Case 3

A 20-year-old man is admitted with a flare of his ulcerative colitis and


treated with intravenous hydrocortisone. About 24 hours following
admission he becomes increasingly drowsy. His Glasgow Coma Scale
(GCS) score drops to 8 and his temperature is 37.5 °C, he has bilateral
papilloedema, he is flexing all his limbs to painful stimuli, all his tendon
reflexes are present and symmetrical and both plantar reflexes are
extensor. He subsequently has a generalised seizure and is intubated and
ventilated.
His blood tests are:
Sodium 148 mmol/L
Potassium 3.1 mmol/L
Urea 10.4 mmol/L
Creatinine 120 Mmol/L
Hb 10.1 g/dL
MCV 79 fL
WCC 16.5 x 107L
Platelets 430 x 107L
ESR 80 mm/h

His CT head with contrast is shown below:

49
NEUROLOGY - QUESTIONS

1 What is the diagnosis?


LJ A Sagittal sinus thrombosis
L J B Bacterial meningitis
ÿ C Intracerebral abscess
U D Intracerebral haemorrhage
ÿ E Cerebral oedema

2 The correct treatment is:


D A Intravenous ceftriaxone
ÿ B Therapeutic anticoagulation with heparin
ÿ C Intravenous dexamelhasone
L D Referral to Neurosurgery Department for evacuation of clot
ÿ E Rehydration with normal saline

.50
NLUROLOCY QUESTIONS

Case 4

A 30-year-old right-handed man attends A&E. Yesterday he went


mountain hiking. Today he noticed a pain in his neck andhis left arm
feels funny.
On examination he has a right partial ptosis and the right pupil is smaller
than the left. The remainder of the cranial nerves are normal. You note
slight loss of dexterity in the left hand; there is no weakness or ataxia, t lis
reflexes are brisker on Ihe left than on the right, both plantar reflexes are
flexor. He has evidence of sensory neglect in the left upper and left lower
limbs.
He has had a CT head which is reported as normal.

1 The best diagnosis is:


ÿ A Vertebral artery dissection
ÿ B Acute cervical disc protrusion
ÿ C Right carotid artery dissection
ÿ D Right middle cerebral artery infarct
ÿ E Right lateral medullary syndrome

2 The most appropriate next investigation is:


ÿ A CT cervical spine
ÿ B MRI brain and cervical spine
ÿ C Percut.ineous a ngiography
ÿ D Plain X-ray of neck
ÿ E Carotid Doppler

3 The most appropriate management is:


ÿ A Aspirin 300 rng stat
ÿ B Thrombolysis with tPA
ÿ C Therapeutic anticoagulation with heparin
ÿ D Referral to a neurosurgeon
ÿ F Analgesia and physiotherapy

51
NEUROLOGY- QUESTIONS

Case 5

A 30-year-old lady is brought to hospital by ambulance. The night


before, she had complainedof nausea andheadache to her husband. She
hadgone to bedand on waking he found that she was confused and
irritable.
On admission she was 37.5 °C; GCS was 8; there was neck stiffness; the
pupils were equal and reactive; there was no papilledema or
haemorrhages; she flexed all four limbs to pain; the reflexes were brisk
throughout; and the plantar reflexes were extensor.
The patient is intubated and ventilated and a CT scan (without contrast)
is shown.

PI

1 The diagnosis is:


ÿ A Bacterial meningitis CD D Intracerebral haemorrhage
ÿ B Venous sinus thrombosis ÿ E Subdural haemorrhage
ÿ C Subarachnoid haemorrhage

2 What complication has occurred?


C A Abscess formation
B Hydrocephalus
CD C Middle cerebral artery (MCA) vasospasm
I I D Malignant MCA syndrome
ÿ E Venous infarction

52
NEUROLOGY QUESTIONS

Case 6

A 55-year-old Indian lady attends Outpatients complaining of fatigue


and muscle pains, particularly affecting her shoulders and thigh muscles
She has difficulty walking upstairs and difficulty rising from a chair.
There is no family history of neuromuscular disorders.
On examination, temporal arteries are normal and non-tender; she has
normal cranial nerve examination; she has a proximal weakness in both
upper and lower limbs and difficulty rising from a squat. Reflexes and
sensory examination are normal.

The CP has helpfully performed a number of blood tests:


Hb 1 1 .0 g/dL
MCV 95 fL
WCC 5.2 x 10y/L
Platelets 420 x 1071.
Sodium 1 40 rnmol/l
Potassium 3.6 mmol/l
Urea 7.0 mmol/l
Creatinine 62 iimol/l
Albumin 37 g/L
ALT 20 U/L
ALP 388 U/L
Bilirubin 20 jtmol/L
Calcium 2.10 mmol/L
Phosphate 0.8 mmol/L
Creatine kinase 305 U/L

1 The diagnosis is:


'
Polymyositis
J A
; ! B
Polymyalgia rheumatica
ÿ C Myasthenia gravis
I ! D Osteomalacia
0 t I lypothyroidism

5.3
NEUROLOGY - QUESTIONS

Case 7

A 25-year-old Peruvian man, who came to this country a year ago, is


referred to you in A&E. He describes abnormal movements of his left
arm. These occur without warning: he describes the arm as stiffening and
then shaking for Ito 5 minutes. He is unable to control the arm during
the episode and it often feels weak for 30 minutes to an hour afterwards.
He does not have any impairment of consciousness during this period.
These episodes are occurring approximately once a week andhe had a
further episode this morning. His CP has started him on carbamazepine.
He complains of painful thigh muscles and on palpation they are tender;
otherwise neurological examination is normal. He is afebrile.

He has had a CT scan which is shown below:

Carbamazepine trough level 22 pmol/L

1 The diagnosis is:


D A Neurocysticercosis
ÿ B Multiple cerebral metastases
ÿ C Cerebral toxoplasmosis
G D Cerebral lymphoma
ÿ E Ascaris lumbricoides

54
NEUROLOGY - QUESTIONS

2 Regarding management of his seizures, you would suggest:


ÿ A Increasing carbamazepine and re-checking levels in a week's lime
D B Give a loading dose of intravenous phenytoin in A&E
L) C Adding sodium valproate to carbamazepine
LJ D Stopping carbamazepine and substitute phenytoin
C E Giving a stal dose of oral diazepam

55
NEUROLOGY QUESTIONS

Case 8

You are asked to review a patient on the ward. He is 35 years oldand


drinks 80 units of alcohol a week. He was admitted 2 days ago with
vomiting andgeneralised tonic-clonic seizures. He was treated with
thiamine, fluid replacement, diazepam andphenytoin. He is now
seizure-free but now complains that he can no longer walk.
On examination he has numerous spider naevi, pupils are equal and
reactive and fundoscopy is normal. He has gaze-evoked horizontal
nystagmus with reduced eye abduction bilaterally. I lis speech is slurred
but he is lucid and orientated. He has bilateral facial weakness and
marked weakness of both upper and lower limbs. His deep tendon
reflexes are depressed throughout and both plantar reflexes are extensor.
You review the current blood tests and those performed on admission:

Admission Current
Sodium 114 139 mmol/L
Potassium 3.3 4.2 mmol/L
Urea 9.0 5.2 mmol/L
Creatinine 100 80 umol/L
Albumin 35 35g/L
AIT 40 42 U/L
ALP 210 200 U/L
vGT 400 390 U/L
Bilirubin 25 24 pmol/L
Calcium 2.30 2.3 mmol/l
Phosphate 0.9 0.9 mmol/L

You request a CT brain which is normal.

1 The diagnosis is:


CD A Guillain-Barre syndrome
CD B Brainstem stroke
CD C Wernicke's encephalopathy
CD D Central pontine myelinolysis
CD L Hepatic encephalopathy

56
NEUROLOGY - QUESTIONS

Case 9

A 50-year-old man is referred to the clinic because of unsteadiness on


walking. He has also had a number of falls. He first noticed his
symptoms 3 months ago and they have been gelling progressively worse;
more recently his speech has become slurred. He has a past history of
hypertension which is treated with bendroflumethiazide(hendrofluazide)
and he smokes 10 cigarettes a day. He drinks 20 units of alcohol a week.
On examinalion ho has sustained horizontal nystagmus in both
directions. He is dysarthric and has marked upper and lower limb ataxia
bilaterally. Power is normal throughout, and the sensory examination is
also normal.

Routine blood tests, including liver function and thyroid function tests
are normal. He has had an MRI brain which is reported as normal with
no evidence of cerebellar atrophy.

A lumbar puncture is performed:


CSF glucose 4.0 mmol/L (serum 6.0 mmol/L)
CSF protein 0.8 g/dL
CSr red cell count 5/mm1
CSF white cell count I lymphocyte/mm'

Matched oligoclonal bands are positive in CSF and serum.

1 The most likely diagnosis is:


D A Alcohol-related cerebellar degeneration
ÿ B Multiple sclerosis (MS)
ÿ C Friedreich's ataxia
CD I) Paraneoplastic cerebellar degeneration
D E Herpes zoster cerebellitis

57
NEUROLOGY QUESTIONS

Case 10

A 60-year-old right-handedlady complains of severe pain in her right


shoulder that started 3 weeks ago. This started Iweek alter receiving
pneumococcal vaccine, which was administered into her right arm.
There was no history of trauma. More recently her pain has improved;
however she has notedthat she is unable to lift her arm above her head.
On examination there is wasting of the deltoid, which is weak, as is the
biceps muscle. The right biceps and supinator jerks are absent; the
remainder of the reflexes are normal. On sensory examination she has
reduced sensation to pinprick over the right shoulder and the lateral
aspect of the arm. Examination of the lower limbs is normal.

1 The most likely diagnosis is:


D A Axillary nerve palsy
ÿ B Cervical disc causing root compression at C7
D C Neuralgic amyotrophy
I I D Neoplastic infiltration of the brachial plexus
D E Avulsion injury of the brachial plexus

58
NEUROLOGY QUESTIONS

Case 11

An 18-year-oldgirl complains of unusual episodes of collapse. Typically


these occur when she laughs. She suddenly feels weak as if she has no
power in her legs and drops to the ground. She is completely alert and
orientated during this time andmakes a rapid recovery. She also
describes an unusual experience in the morning of waking and not being
able to move. She is anxious about her symptoms; she has not been
sleeping well at night and feels tired.
Cardiovascular and neurological examinations are normal.

1 The diagnosis is:


ÿ A Narcolepsy
B Non-epileptic attack disorder
Cj C Temporal lobe epilepsy
D Hypoglycaemia
C7] E Hypothyroidism

59
NEUROLOGY - QUESTIONS

Case 12

/\ 60-year-old man complains of progressive difficulty with walking over


the last 3 months and, more recently, a dry mouth, difficulty swallowing
and difficulty passing urine.
On examination he is generally thin; there are no fasciculations. He has
clubbing and a fatiguable proximal weakness in both upper and lower
limbs. His tendon reflexes are present but depressed, plantar reflexes are
flexor. Sensory examination and co-ordination are normal.

A number of blood tests are performed:


Hb 10.5 g/dL
MCV 95 t'L
WCC 5.2 x 107L
Platelets 420 x 1 0'VI
Sodium 141 mmol/l
Potassium 3.8 mmol/l
Creatinine 62 pmol/L
tSR 40 mm/li
Creatine kinase 1 90 U/L
Anti-ACh receptor Ab Negative

1 The diagnosis is:


CD A Polymyositis
CD B Motor neurone disease
CD C Myasthenia gravis
CD D Lambert-Eaton myasthenic syndrome
CD E Cervical spondylosis

60
NEUROLOGY - QUESTIONS

Case 13

A 20-year-old right-handedlady is referred by her CP with unusual


episodes. These have occurredsince the age of 18 but have recently
become more frequent; the last was a week ago. They usually begin with
a strange epigastric sensation which rises up. She feels fearful and distant
from her surroundings. Her husbanddescribes her as being distant and
says that she sometimes picks at objects around her (there may also be
chewing movements). She is aware other heart beating strongly. These
episodes last for up to 10 minutes, although she may be confused for up
to an hour afterwards. She had one febrile convulsion as a child.

Neurological and cardiovascular examination is normal.

An EEC is performed which is normal and a CT head is normal.

1 The most likely diagnosis is:


t 3 A Panic disorder
B Phaeochromocytoma
. . C Temporal lobe epilepsy
D Absence seizures
( I E Cardiac arrhythmia

2 What should the patient be told about driving?


( I A She did not lose consciousness and so can continue to drive
CD B She should not drive for 6 months from her last episode
CD C You advise her not to drive and that she should inform the DVLA
CD D You will inform the DVLA of these episodes
CD E She can continue to drive until investigations are complete

61
NEUROLOGY - QUESTIONS

Case 14

A 40-year-old man attends A&E. He complains of progressive weakness,


paraesthesiae and numbness that began in his feet, spreading up both
legs and then involved both his hands over 3 days, associated with fever.
Today he has had difficulty passing urine. He has a past history of
intravenous heroin use.
On examination he is unkempt, he has multiple injection sites, he has a
temperature of 38 °C and a soft systolic murmur is heard at the apex. He
is alert and orientated. He complains of pain on passive neck movement
and neck stiffness is noted. He has a severe flaccid weakness of both
upper and lower limbs and his tendon reflexes are absent. Plantar
reflexes are extensor. He complains of reduced sensation to pinprick in
the lower and upper limbs, and up to just below the level of the clavicle
bilaterally and to the same level on the posterior chest wall. Vibration
sense and joint-position sense are also reduced in this distribution.

An urgent MRI cervical spine is performed.

62
NEUROLOGY - QUESTIONS

1 The most likely diagnosis is:


I I A Anterior spinal artery infarction
I 1B Transverse myelitis
iI C Cervical disc prolapse
LJ D Spinal abscess
D E Vertebral metastatic deposit
You organise basic blood tests, a septic screen, ECG and chest X-ray.

2 Your next management step should be:


LJ A Antituberculous treatment
[ J B Intravenous immunoglobulin treatment
LJ C Broad-spectrum antibiotics and intravascular dexamethasone
D Broad-spectrum antibiotics and neurosurgical referral
ÿ E HIV testing

63
NEUROLOGY - QUESTIONS

Case 15

A 50-year-oldItalian man developed a left foot drop 2 months ago and


then 2 weeks ago notedreducedgrip strength in his right hand
associated with pain in the fingers. He describes a fleeting rash on his
feet. He has chronic hepatitis C. His current medication is
bendroflumethiazide (bendrofluazide) 2.5 mg.
On examination he has weakness of the right first dorsal interosseus and
abductor digili minimi, as well as ankle dorsiflexion and eversion on the
left. There is a patch of reduced sensation to pinprick on the dorsum of
the left foot and on the right little finger and ring finger. Vibration sense
and proprioception are preserved. Reflexes are all present.

Urinalysis reveals protein +, blood ++ and no bilirubin. A number of


blood tests are performed:
Hb 1 1.6 g/dl
MCV 95 fL
wcc 5.2 x 107L
Platelets 420 x 1071
Sodium 142 mmol/L
Potassium 3.9 mmol/L
Creatinine 1 50 pmol/L
ESR 50 mm/h
Creatine kinase 1 80 U/L
Albumin 33g/L
ALT 160 U/L
ALP 150 U/L
vGT 200 U/l
Bilirubin 24 jimol/l
ANA Negative
ANCA Negative
Prothrombin time 1 7 s
Plasma glucose 5.4 mmol/L
CRP 40 mg/L

1 What is the pattern of the neuropathy?


0 Generalised sensorimotor neuropathy
A
0 Right radial, left common peroneal
B
O Right ulnar, left tibial
C
I J D Right median, left sciatic
I , E Right ulnar, left common peroneal

64
NEUROLOGY - QUESTIONS

2 The most likely diagnosis is:


D A Vitamin B,2 deficiency
CJ B Chronic inflammatory demyelinating polyneuropathy (CIDP)
ÿ C Cryoglobulinaemia
ÿ D Paraneoplastic neuropathy
CD E Wegener's granulomatosis

65
NEUROLOGY QUESTIONS

Case 16

A 65-year-old man was diagnosed with idiopathic Parkinson's disease


5 years ago when he developed a rest tremor and stiffness of the left arm.
He was commenced on levodopa therapy J years ago when his walking
slowedand he says this was helpful. He was admitted to hospital after
having a fall and fracturing his hip. This was successfully repaired but the
physiotherapists feel that he is much slower to mobilise than they would
expect. Ilis current medication is co-careldopa 125 mg qds, tramadol 50
mg bdand metoclopramide 10 mg ids.
On examination he has poverty of facial expression, his speech is soft
and he has difficulty swallowing. He has marked rigidity in all four limbs
a rest tremor, more marked in the left arm, and is very bradykinetic. He is
able to stand with difficulty but cannot walk. He has been noted to have
a labile blood pressure since admission.

1 You should:

CD A Gradually increase levodopa dose


0 B Perform a full septic screen
Q C Add an anticholinergic agent such as trihexyphenidyl (benzhexol)
0 D Change metoclopramide to domperidone
0 E Inform the patient that he has multiple systems atrophy

66
NEUROLOGY - QUESTIONS

Case 17

An 80-year-old man is admitted from a nursing home. He has a 3-day


history of headache and today has become increasingly confused.
On examination he has a temperature of 38 °C and has neck stillness. He
has a CCS of 12 on admission and has a witnessed generalised
tonic-clonic seizure.

A CT is performed which is normal. A lumbar puncture is performed. The


opening pressure is 24 cmH.O and the CSF is cloudy:

CSF glucose 3.5 mmol/L


CSF protein 0.8 g/dL
CSF red cell count 50/mnT
CSF white cell count 75 lymphocytes, .5 polymorphs/mnV
Gram stain Gram-positive rods
Serum glucose 6.0 mmol/L

1 The treatment of choice is:


( J A Benzylpenicillin
IJ B Cellriaxone and clarithromycin
ÿ C Ceflriaxone and ampicillin
ÿ D Aciclovir
LJ E Antituberculous therapy

67
NEUROLOGY - QUESTIONS

Case 18

The photograph shows the eyes of a 60-year-old man who developed


diplopia, a complete left ptosis over a number of days and then a right
hemiparesis.

1 Where is the lesion?


ÿ A Left internal capsule
ÿ B Left frontal lobe
ÿ C Left cerebellopontine angle
ÿ D Left orbit
ÿ E Left midbrain

68
NFUROI OC Y - QUESTIONS

Case 19
A 50-year-oldman who lives alone is found collapsed at home; he is in
respiratory failure and is intubated by paramedics. His admission bloods
are shown below. On ITU he is treated for pneumonia but when his
sedation is reducedhe is noted to have facial and limb weakness andhe
is difficult to wean off the ventilator. Neurophysiology is performed on
ITU and the figure shows the results of repetitive nerve stimulation.
Hb 1 3.8 g/dL
VVCC 9.3 x 107L
MCV 99 II
Platelets 129 x 107L
Sodium 133 mmol/L
Potassium 3.4 mmol/L
Bilirubin 12 pmol/L
ALT 50 U/L
yGT 400 U/L
Cortisol 1003 nmol/L
mV RECALLING NUMBER : || 1
'

5 S

STIM . MODE : >1:1:11.1 I SINGLE

AMI'
DISCI

1 The diagnosis is:


CD A Critical-illness neuropathy
CD B Motor neurone disease
CD C Myopathy
CD D Myasthenia gravis
CD E GuiIlain-Barre syndrome

69
NEUROLOGY - QUESTIONS

Case 20

A 35-year-old lady gives a 2-day history of severe headache, fever and


nausea. There is no history of associated neck stiffness or photophobia.
On the morning of admission she hadalso developed ocular pain and
swelling bilaterally and diplopia.
On examination she had a temperature of 39 °C; she was alert and
orientated. Her face is shown below. Pupillary responses were sluggish.
Her visual acuity was 6/6 bilaterally; fundoscopy revealed dilated retinal
veins and early papilledema. She had a complete ophthalmoplegia
bilaterally. Pinprick sensation was reduced over the forehead.

1 What is the diagnosis?


ÿ A Thyroid eye disease D Cerebral abscess
ÿ B Sagittal sinus thrombosis E Pituitary apoplexy
ÿ C Orbital cellulitis ÿ F Cavernous sinus thrombosis

2 Give the two most important steps in the management:


ÿ A Arterial blood gasses
ÿ B Sinus X-rays
ÿ C Subcutaneous octreotide
ÿ D CT head with contrast
ÿ E Transarterial angiogram
ÿ F Blood cultures
ÿ G Thyroid antibodies
ÿ H MRI pituitary and hypothalamus
ÿ I Lumbar puncture

70
OPHTHALMOLOGY - QUESTIONS

Case 1

Study the Goldmann visual field test below.

1 Where is the pathological lesion?


ÿ A Left occipital cortex
ÿ B Left temporal lobe (Meyer's loop)
ÿ C Right occipital cortex
ÿ D Right optic nerve
ÿ E Right parietal lobe

73
OPHTHALMOLOGY- QUESTIONS

Case 2

The surgical registrar calls you at 1 1pm to see a 72-year-old Chinese


lady. She is under the general surgeons, being investigated for possible
bowel obstruction because of severe nausea and vomiting. She is
vomiting every half-hour and complains of progressive visual loss in the
right eye during the course of the evening as well as redness andpain.
She noticedthat for the preceding few days she has seen halos round
street lamps. She does not smoke, only drinks rarely and lives on her
own except that recently she has purchased a cat.
You do not have a Snellen or a )aeger chart but on using a magazine, her
acuity is better in the left than in the right eye. On inspection there is a
shallow anterior chamber, a semidilated, fixed pupil and the appearance
is shown below. There is a little clear discharge.
Her blood results were:
FBC Normal
ESR 29 mm/h
Sodium 131 mmol/L
Potassium 3.2 mmol/L
Bicarbonate 34 mmol/L
Chloride 99 mmol/L
Urea 6.9 mmol/L
Creatinine 110umol/L

74
OPHTHAI MOLOGY QUESTIONS

1 Which of the following two statements are true?


0 A An oral antihistamine and hydrocortisone eye drops should be
prescribed
0 B The findings are consistent with acute angle-closure glaucoma
n C The findings are consistent with anterior uveitis
ÿ D She must see an ophthalmologist first thing in the morning
0 E Topical antibiotic drops should be commenced immediately once a
swab is taken
0 F This condition is self-limiting within 4-14 days, with no specific
treatment
0 G A dilated fundus examination should be undertaken immediately
0 H Acetazolamide 500 mg should be administered intravenously once
the diagnosis is confirmed
0 I Prednisolone, 1 mg/kg should be given once the diagnosis is
suspected
[ ) ) Intravenous antibiotics to cover Staphylococcus aureus,
Streptococcus pneumoniae and Haemophilus influenzae must be
commenced once swabs and blood cultures are taken

75
OPHTHALMOLOGY - QUESTIONS

Case 3

A 65-year-old overweight Asian lady was referred to Ophthalmology


Outpatients with general malaise and a gradual reduction in vision of
3 months' duration, affecting both eyes.
Aided Snellen visual acuity was recorded as 6/1 8 right eye, 6/1 2 left eye,
both eyes improving to 6/9 with a pinhole. Dilated examination revealed
bilateral, moderate nuclear sclerosis, and widespread retinal changes as
shown below.

Her lipids were checked by her CP 4 months before:


Cholesterol 6 mmol/L
HDL 0.79 mmol/L
LDL 4.0 mmol/L
Fasting triglycerides 4.6 mmol/L

76
OPHTHALMOLOGY - QUESTIONS

1 Which of the following is the most likely underlying diagnosis?


f ] A Ankylosing spondylitis
1 !
R Diabetes mellitus
C] C Hyperlipidaemia
i .
_ D
E
Multiple sclerosis
Sarcoidosis

77
OPHTHALMOLOGY - QUESTIONS

Case 4

A 65-year-old man presents to A&E with reducedvision in the right eye,


noticed while taking a photograph.
On examination he has a right afferent pupillary defect and the following
appearance on fundoscopy

1 Which two of the following should be undertaken


ÿ A Blood pressure measurement
ÿ B Thyroid function
ÿ C CT orbits
ÿ D Chest X-ray
ÿ E Toxoplasma serology
ÿ F Referral for fluorescein angiography
ÿ C Carotid Doppler
ÿ H Referral for intraocular pressure measurement
ÿ 1 Counsel for an HIV test
ÿ 1 MRA carotid circulation

78
RHEUMATOLOGY - QUESTIONS

Case 1
A 52-year-old man presented with an 18-month history of malaise,
anorexia, arthralgia and weight loss. He felt this was attributable to a
persistent cold with a chronic dry cough and a blocked nose.
On examination he was 1.88 m tall and weighed 58 kg. He had a
purpuric rash on the dorsum of his feet, extending to mid-shin. His
musculoskeletal examination revealed no synovitis and was otherwise
normal, except for his inability to actively dorsiflex his right foot.
Investigations showed:
HB 1 0.4 g/dL Urea 3.7 mmol/L
MCV 86 fL Creatinine 1 18 umol/L
WCC 10.4 x 107L Antibodies:
Platelets 497 x 10"/L Anti-proteinase-3 Positive
ESR 76 mm/h Anti-myeloperoxidase Negative
Sodium 1 37 mmol/L Urine microscopy Red cell cast
Potassium 4.4 mmol/L

81
RHLUMATOI OCY - QUESTIONS

1 What is the most likely diagnosis?


[ '
Wegener's granulomatosis
A
[ 3Microscopic polyangiitis
B
I ] C Rheumatoid arlhrilis
LI D Secondary lung metastases
L_l E Goodpasture's syndrome

82
RHEUMATOLOGY - QUESTIONS

Case 2

A 44-year-old man presents with pain and swelling of the small joints of
his hands with associated difficulty in gripping small objects, particularly
with his dominant left hand. It has been getting gradually worse over the
last 2 years. He has been feeling tired and has lost { stone in weight.
Investigations requested by the GP showed:
Hb 11 .6 g/dL
MCV 86 fL
WCC 10.4x10"/L
Platelets 427x107L
ESR 84 mm/ h
Antibodies:
Rheumatoid factor Positive, 1/80
ANA Negative
His hand X-rays are shown below:

83
RHEUMATOI OGY - QUESTIONS

1 What is the most likely diagnosis?


ÿ Osteoarthritis
A
I Rheumatoid arthritis
I?
0 C Psoriatic arthritis
0 D Gout
0 E Polyarticular septic arthritis

84
RHEUMATOLOGY - QUESTIONS

Case 3
A 19-year-old accounts clerk presented with pain in her left knee and
ankle. She hadtailed to gain weight since the age of 14. She also suffered
from frequent aphthous ulcers.
Her investigations show:
Hb 12.7 g/dL
WCC 5.0 x 107L
Platelets 337 x 107L
U&Es Normal
Corrected CaJ' 3.1 mmol/L
Phosphate 0.6 mmol/L
ALP 942 U/L
Ferritin 4 pg/L
B,i 100 ng/L
Red cell folate 103 pg/L
PTH 50 pmol/L

85
RHEUMATOLOGY - QUESTIONS

1 What is Ihe diagnosis?


CD A Whipple's disease
CD B Coeliac disease
CD C Localised osteoporosis
CD D Enteropathy: arthritis
CD E Primary hyperparathyroidism

86
RHEUMATOI OCY QUEST IONS

Case 4

A 42-year-old machine operator, who has had Raynaud's phenomenon


for the last 7 years,presents with an impairedgrip, puffy swollen fingers
and difficulty swallowing.
Extractable nuclear anybodies (INA) are strongly positive with a
centromeric pattern.
Extractable antinuclear antibodies are as follows:
Topoisomerase I (Scl-70) Negative
Centromere Positive
Jo I Negative
U1-RNP Negative

1 What is the most likely diagnosis?


_
I Hand-arm vibration syndrome
A
M Limited cutaneous systemic sclerosis
13
D C Diffuse cutaneous systemic sclerosis
ÿ D Systemic lupus erythematosus (SLE)
O E Mixed connective tissue disease

87
RHEUMATOLOGY - QUESTIONS

Case 5

A 79-year-old woman presented to the Rheumatology Clinic with a


history of sudden-onset low hack pain while doing some digging in the
garden. Her X-ray revealedanterior wedge fractures at L2 and L4. She
sustained a Colles' fracture to her right wrist 2 years ago following a fall
from a standing height.
Her DFXA result is shown:

Region BMD g/cm2 T-score Z-score


L2-L4 0.857 -2.9 -1.2

1 What is the most accurate description of her bone mineral density?


ÿ A Normal
( )
B Osteopenia
0 C Osteomalacia
0 D Osteoporosis
0 E Severe osteoporosis

88
RHEUMATOLOGY - QUESTIONS

Case 6

A 68-year-old woman has been recently started on 5 mg prednisolone for


rheumatoidarthritis. It is planned that she will stay on this treatment as
previous disease-modifying antirheumatic drugs (DMARDS) havebeen
either ineffective or have had adverse effects. She expressed concerns
about her bones as she has a family history of osteoporosis. She has
never previously been on steroids. She entered puberty at the same time
as her peers and entered the menopause aged 52. She has had two
children. She has always done regular weight-bearing activity. Her
dietary calcium intake has always been low. She drinks no alcohol and
does not smoke.
Her DEXA bone scan showed a T-score of -2.2 at the hip and -1.7 at the
spine.

1 What is the most appropriate management?


O A l ifestyle advice
D B Lifestyle advice plus repeat BMD in I -3 years if steroids continued
D C Raloxifene
D D Bisphosphonate
ÿ E Recombinant parathyroid hormone (PTH)

8')
RHEUMATOLOGY - QUESTIONS

Case 7

A 52-year-oldman with severe exfoliative psoriasispresented with an


acutely painful, swollen left ankle. He had had intermittent episodes of
pain and swelling affecting his hands, elbows, knees and feet over the
preceding 5 years.
On examination he had soft tissue swelling over his left index and
middle and right index proximal interphalangeal (PIP) joints, though
there was no synovitis. In addition to psoriatic plaques at his elbows,
there was a firm nodule within a prominent olecranon bursa on the right.
The left ankle was tender, with overlying erythema and pitting oedema.
Movement was restricted and walking was very painful.

An X-ray of his hands is shown:

1 What is the most likely diagnosis?


ÿ A Osteoarthritis
ÿ B Rheumatoid arthritis
ÿ C Psoriatic arthritis
ÿ D Polyarticular septic arthritis
C E Gout

90
RHEUMATOLOGY QUESTIONS

Case 8

/A 62-year-old woman is admitted through A&E. She states that she had a
lot ofjoint problems in her thirties and forties but has been relatively
pain-l'ree for the last 20 years. She now presents with a painful, swollen
right knee.
On examination, she has swan-neck and boutonniere deformities in both
hands, with subluxation and reduced range of movement in both wrists.
Both forearms have firm, subcutaneous nodules just distal to the elbow.
The right knee is hot and red with a large effusion. The range of
movement is markedly reduced, limited by pain. The left knee has some
mild crepitus but no effusion. She has clawing of the toes with callosities
on the soles of both feet. A metatarsophalangeal squeeze is not painful.

1 What is the most important first investigation in the management of this


patient?
ÿ A Full blood count
ÿ B Rheumatoid factor
ÿ C Serum uric acid
ÿ L) Blood cultures
ÿ E Synovial fluid analysis of the right knee effusion
ÿ r X-ray hands and feet
ÿ C, X-ray right knee

91
RHEUMATOLOGY QUESTIONS

Case 9
A 31-year-old man has a 3-month history of pain and swelling of his
hands, knees, right ankle and feet, with morning stiffness lasting 4 hours.
He also describes shoulder pain and right wrist pain. He has had to stop
work as a joiner and currently needs help with washing and dressing.
On examination he has very active synovitis in many
metacarpophalangeal (MCP) and proximal interphalangeal (I'll') joints,
right wrist, both knees, right ankle and metatarsophalangeal (MTP) joints.
He has a reduced range of movement in both shoulders. There are no
rashes, nail changes or subcutaneous nodules.

Investigations are as follows:


I lb 1 1 .8 g/dL
MCV 91 fL
WCC 6.4 x 10"/L
Platelets 497 x 10"/L
ESR 76 mm/h
U&Es Normal
LFTs Normal
CRP 64 mg/L
Rheumatoid factor Positive (1/4096)

X-rays of the hands and feet arc normal.

1 What drug treatment would be most appropriate at this stage?


J A Traditional non-steroidal anti-inflammatory drug
CD B COX-2 inhibitor
CD C Methotrexate
CD D Hydroxychloroquine
ÿ E TNF-a blocker

92
RHEUMATOLOGY - QUESTIONS

Case 10

A 22-year-old woman presented with a 2-day history of a severely


painful left wrist. There was no preceding trauma. Other than feeling hot,
she was otherwise well.
On examination, she had a temperature of 37.9 °C. She had a hot, tender
swelling over the dorsum of the left wrist, extending over the dorsum of
the hand to, but not including, the metacarpophalangeal joints. She was
also found to have a pustular rash over her hands and feet. Plain X-rays
were unremarkable.

1 What is the most likely diagnosis?


LJ A Behcet's syndrome
CD B Lyme disease
ÿ C Acute rheumatic fever
O D Disseminated gonococcal infection
ÿ E Reactive arthritis

93
RHEUMATOLOGY - QUESTIONS

Case 11

A 55-year-old smoker with rheumatoid arthritis presented with a


6-month history of fever, anorexia, weight loss and localisedback pain.
He admitted to an alteration in both bowelandbladder control. He was
on 10 mg methotrexate per week as his only disease-modifying treatment
but hadhad eight intra-articular steroid injections into his knees over the
last 4 years.
His spinal X-ray is shown below:

1 What is the most likely diagnosis?


ÿ A Osteoporosis
D B Spinal tuberculosis
CD C Vertebral metastases
ÿ D Haemangioma
CD E Multiple myeloma

94
RHEUMATOLOGY - QUESTIONS

Case 12

A 38-year-old woman presented with a 2-year history of diffuse ache


aroundher shoulder and hip girdles. She felt weak, though on clinical
testing her muscle strength was normal.
Investigations showed:
ESR 16 mm/h
Creatine kinase 480 U/L

1 What is the next most useful investigation to establish the cause of her
symptoms?
ÿ A Thyroid function
ÿ B Aspartate aminotransferase
ÿ C Muscle biopsy
ÿ D Electromyography (EMG)
ÿ E MRI of thighs

95
RHEUMATOLOGY- QUESTIONS

Case 13

A 74-year-old woman presented with a gradual onset of pain and


stiffness in the shoulder girdle and neck over a period of 6 months. The
pain sometimes woke her from her sleep. The stiffness was worse in the
morning, lasting roughly 2 hours. She experienced pain on chewing.
On examination the patient was depressed with a blunted affect. The
shoulders were tender; there was no axial tenderness; the elbows were
normal; shoulder abduction was limited to 70° bilaterally, both active
and passive; there was no swelling or tenderness at the sternoclavicular
joints; hip flexion was full, though painful, and there was a moderate left
knee effusion. She was unable to stand from a chair unaided without the
use of her arms. Formal muscle strength testing was not possible because
of pain, though her distal muscle strength was normal.

Investigations were as follows:


Hb 12.1 g/dL Calcium 2.18 mmol/L
MCV 88 fL Phosphate 0.9 mmol/L
WCC 7.4 x 107L Albumin 36g/L
Platelets 1 77 x 107L ALP 120 U/L
ESR 86 mm/h Creatine kinase 95 U/L
U&Es Normal Rheumatoid factor Negative

96
RHEUMATOLOGY QUESTIONS

1 What is the most likely diagnosis?


D A Osteomalacia
D B Polymyositis
D C Rheumatoid arthritis
D D Fibromyalgia
D E Polymyalgia rheumatica

97
RHEUMATOLOGY - QUESTIONS

Case 14

A 34-year-old man presents with back pain and alternating buttock pain.
It is worse through the night and in the morning, when he struggles to get
out of bed. Recently he has had significant chest pain. He has suffered
from two self-limiting episodes of a red left eye. His father suffered from
psoriasis.

On examination there is some swelling with tenderness over the right


sternoclavicular joint. He is tender over the right sacroiliac joint. He is
able to touch his toes.

1 What is the most likely diagnosis?


ÿ A Ankylosing spondylitis
ÿ B Degenerative disc disease
ÿ C Lumbar spondylosis
ÿ D Diffuse idiopathic skeletal hyperostosis
ÿ E Reiter's syndrome

98
RHEUMATOLOGY - QUESTIONS

Case 15

A 24-year-old man presented with pain in the sole of his right foot and
behindthe right ankle. He was finding it difficult to walk in the mornings
as it was agony to put his foot to the floor. This sharp pain eased through
the morning. In addition, he had pain and swelling of the left knee. His
upper limbs were asymptomatic. He had not had any chest pain,
breathlessness, gastrointestinal symptoms or rashes, though he hadhad
some mild dysuria 4 weeks earlier lasting 2 days.
Urinalysis is normal. Other investigations showed:
Hb 16.2 g/dL
MCV 91 fL
wcc 1 4.2 x 109/L
Platelets 504 x 10'VL
FSR 54 mm/h
Synovial fluid analysis (left knee):
WCC 6000 cells/mm3
Gram stain Negative
Culture Negative

1 Which diagnosis is most likely?


ÿ A Gout
D B Gonococcal arthritis
ÿ C Reactive arthritis
ÿ D Achilles bursitis
D I: Psoriatic arthritis

99
RHEUMATOLOGY - QUESTIONS

Case 16

A 65-year-old woman presented with right shoulder pain. She described


it as a deep, aching pain, radiating to the neck, with associated stiffness.
The pain was worse with any shoulder movement, and worse in bed at
night. Her only other musculoskeletal symptom was of thoracic back
pain.
On examination she had a restricted range of right shoulder movement
in all planes, with active and passive abduction to only 60°. She was also
found to have fixed flexion deformities of all her fingers, though no
impairment of hand function.

100
RHEUMATOLOGY -QUESTIONS

1 What is the unifying diagnosis?


CD A Ankylosing spondylitis
ÿ B Diabetes mellitus
CD C Cervical spondylosis
CD D Limited cutaneous systemic sclerosis
CD E Adhesive capsulitis

101
RHEUMATOLOGY - QUESTIONS

Case 17

A 72-year-old woman is having increasing difficulty getting up from a


chair. She finds it harder to stand once she has been sitting in a chair for
a long period of time. Her walking distance is gradually deteriorating and
she needs to stop to sit down on more than one occasion when walking
aroundher local supermarket.
On examination she is overweight. There are firm swellings over most
proximal interphalangeal (PIP) joints and the right index distal
interphalangeal (DIP) joint. There is no rash visible and her nails are
normal. Her hand function is normal. There is a reduction of external
rotation at the right hip, though full flexion. She has weakness of hip
flexion bilaterally, worse on the left.

1 What is the most likely diagnosis?


L 1 A Hypothyroidism
Q B Dermatomyositis
O C Rheumatoid arthritis
CD D Psoriatic arthritis
ÿ E Osteoarthritis

102
RHEUMATOLOGY - QUESTIONS

Case 18

A 62-year-old man presents with left groin pain that is worse on weight-
bearing. He does not describe any inactivity stiffness. The pain can also
be present at night and regularly disturbs his sleep. He has no other joint
symptoms. His past medical history includes chronic obstructive
pulmonary disease (COI'D) for which he has recently completeda
reducing dose of steroids, type 2 diabetes mellitus and carcinoma of the
prostate.
His left hip X-ray is shown below:

1 What is the most likely diagnosis?


ÿ A Osteonecrosis
ÿ B Osteoarthritis
ÿ C Bony metastases
O D Osteomyelitis
ÿ E Osteoporosis

103
RHEUMATOLOGY - QUESTIONS

Case 19

A 64-year-old woman with rheumatoid arthritis (KA) presented with a


6-month history of tingling in her right hand that was worse in bed at
night. She had noticed that she was starling to drop objects.
On examination she had synovitis in her right middle PIP joint and her
right wrist, with mild ulnar deviation at the MCPs in both hands. The
thenar and hypothenar eminences in both hands were symmetrical,
though slightly wasted. She had altered sensation in the thumb, index
and middle finger on the right only. There was weakness of the abductor
pollicis brevis and opponens pollicis on that side. Her neck movements
were reduced by about 50%. The remainder of her neurological
examination was normal.

1 What is the most likely diagnosis?


ÿ A Carpal tunnel syndrome
O B Syringomyelia
ÿ C Active rheumatoid arthritis
ÿ D C6 radiculopathy
ÿ E Motor neurone disease

104
RHEUMATOI OC.Y - QUESTIONS

Case 20
A 29-year-old woman attended A&E following a fall onto an outstretched
hand. She sustained an undisplaced wrist fracture that was treated by
immobilisation in a cast. Shortly after removal of the cast, she developed
a severe pain in that limb, worse with dependency.
On examination the limb was warm, red and oedematous. It was
diffusely tender with allodynia. It was not possible to assess muscle
strength due to the pain.

1 What is the most likely diagnosis?


CD A Axillary vein thrombosis
ÿ B Cellulitis
CD C Carpal tunnel syndrome
CD D Complex regional pain syndrome type I
CD E Osteomyelitis

1 05
RHEUMATOLOGY - QUESTIONS

Case 21

A 43-year-old man who has hadrheumatoidarthritis for 12 years


presented with recent weight loss andperipheral oedema. Ile hadalso
recently noticed some rectal bleeding. Current medication was
sulfasalazine Ig hdand diclofenac 50 mg bd.
On examination his pulse was 88 bpm and regular, blood pressure
168/94 mmHg, heart sounds normal. There were bibasal fine inspiratory
crepitations and pitting oedema up to the knees with some sacral
oedema. The oral cavity was normal, the abdomen was soft and
non tender, the tip of the spleen was just palpable, and a rectal
examination found a trace of bright-red blood on the glove. He had
chronic deformities of rheumatoidarthritis in the hands, with moderate
synovitis at the metacarpophalangeal (MCP) joints. There were no
nail-fold infarcts, both wrists were fused.

Invest igat ions showed:


Hb 9.5 g/dL
MCV 81 fL
WCC 9 x 10'VL
Platelets 249 x 10TL
FSR 143 mm/h
CRP 312 mg/L
Urea 35.2 mmol/L
Creatinine 872 umol/L
Total protein 51 g/L
Albumin 16 g/L
Calcium 1 .57 mmol/L
Phosphate .3.2 mmol/L
PTH 20 pmol/L
Urinary protein 1 2 g/day

1 What is the unifying diagnosis?


O A Renal osteodystrophy
0 B Rheumatoid vasculitis
0 c: Adverse effects of diclofenac
I J D Amyloidosis
0 L Felty's syndrome

106
RHEUMATOLOGY - QUESTIONS

Case 22

A 24-year-old woman presented feeling generally unwell, with joint


pains and myalgias. She was on the combined oral contraceptive pill and
minocycline.
Examination found a raised, erythematous rash on her ears, cheeks and
chin. There was some mild synovitis at her right wrist. Urinalysis
revealed protein ++.

Investigations were as follows:


lib 1 1.7 g/dL
WCC 4.2 x 1 0"/L
Neutrophils 80%
Platelets 133 x 1071
U&Es Normal
ANA Positive, 1/10,000

Exlractable nuclear antibodies:


Ro/SS-A Negative
La/SS-B Negative
IJ1-RNP Positive
Sm Positive
Jo-1 Negative
Scl-70 Negative

1 What is the most likely diagnosis?


0 A Psoriatic arthritis
0 B Mixed connective tissue disease
0 C Systemic lupus erythematosus
O D Drug-induced lupus
0 E Sarcoidosis

107
RHEUMATOLOGY - QUESTIONS

Case 23

You have seen a 68-year-old man who presented with pain and swelling
in his right knee.
Investigations show:
WCC 14.6 x 107L
ESR 52 mm/h
Corrected Ca'* 2.40 mmol/L
Phosphate 1 .2 mmol/L
ALP 120 U/L
Uric acid 0.48 mmol/L
Cholesterol 6.8 mmol/L
Joint aspiration Cloudy yellow fluid, low viscosity

1 What is the most likely diagnosis?


ÿ A Osteoarthritis
ÿ B Basic calcium phosphate arthropathy
CD C Gout
D Calcium pyrophosphate dihydrate deposition disease
ÿ E Cholesterol emboli

108
HAEMATOl OGY - ANSWERS

Chapter One Answers

Case 1
1 C Split bilirubin
G Direct Coombs' test (DCT)
A Blood film
The bloods suggest a haemolytic anaemia. The split bilirubin will
confirm an unconjugated hyperbilirubinaemia that is consistent with
haemolysis. The DCT will indicate if it is immune or non-immune in
nature. The film will then give a clue to the aetiology.

2 A A membrane disorder, eg hereditary spherocytosis (HS)


B G6PD deficiency (class I)
I Paroxysmal nocturnal haemoglobinuria (PNH)
A significant haemoglobinopathy is excluded by the I lb electrophoresis.
A cholecystectomy in her late twenties is suggestive ol a chronic
haemolytic anaemia that led to pigment gallstone formation.
The differential diagnosis is therefore a red-cell membrane defect, an
enzyme deficiency or an unstable haemoglobin, which are all inherited;
or paroxysmal nocturnal haemoglobinuria, which is an acquired defect
of red cells, white cells and platelets which makes then susceptible to
chronic destruction by complement. PNH is a disease of young adults
but can occur in children and older people. It has a close association
with aplastic anaemia and there is an increased incidence of thrombosis.
Class 1 G6PD deficiency occurs in rare individuals who have mutations
within the gene that encodes G6PD, leading to such severe quantitative
or qualitative deficiency of G6PD that they have a chronic
non-spherocytic haemolytic anaemia. The majority of individuals with
G6PD deficiency, however, have class 2 or .3 (milder) deficiency which
leads to intermittent haemolysis on exposure to one of three triggers -
fava beans, infection or drugs.
The enlarged spleen is a clue, as I IS causes mild to moderate
splenomegaly in virtually all patients. Splenomegaly is usually not seen
in enzyme defects and would only be present in PNH if there had been a
portal vein or hepatic vein thrombosis leading to portal hypertension.
Mild splenomegaly can be seen with an unstable haemoglobin.

11I
HALMATO LOGY - ANSWERS

3 A Hereditary spherocytosis
The lilm shows numerous spherocyles with some polychromatic cells.
The spherocytes are perfectly circular with no central pallor. There are
two polychromatic cells which appear more purplish and in this picture
appear indented.

The presence of spherocytes and polychromasia means Ihere are two


possible diagnoses - HS or All IA. The likely diagnosis is HS because of
the gallbladder removal in her 20s suggesting chronic haemolysis. Mild
splenomegaly can be seen in both conditions. A DCT would differentiate
the two. It would be positive in AIHA and negative in HS.

Case 2
1 G Hb electrophoresis
The bloods suggest a haemolytic anaemia with increased reticulocytes,
raised bilirubinand LDH. The DCT is negative, so it is a non-immune
haemolytic anaemia. The fact that she says she has always been
jaundiced suggests it is an inherited haemolytic anaemia. This means
that it is either a membrane defect, such as hereditary spherocytosis or
hereditary elliptocytosis; an enzyme defect, such as G6PD or pyruvate
kinase deficiency; or a haemoglobinapathy, such as thalassaemia or a
sickling disorder.
Sickle cell disease is the only disease in the above list that is associated
with an increased risk of stroke. There is also a clue in the elevated white
cell count and platelet count, which are commonly seen in sickle cell
disease because of splenic infarction, which leads to splenic atrophy. A
flow murmur secondary to a hyperdynamic circulation commonly
occurs. The Hb electrophoresis will show mainly HbS with a varying
quantity of HbF. The higher the HbF, the milder the clinical phenotype
usually is.
NB The patient was unaware of the diagnosis because her parents did
not want to tell her for fear she would be stigmatised.

2 D Exchange blood transfusion


The immediate management is a euvolaemic exchange blood transfusion
to bring the HbS % down to 30% in adults and 20% in children. This can
be done manually or by machine. The HbS % should remain at these
levels indefinitely by regular exchanges. There is no currently accepted
policy for when exchanges can be ceased. Children and adults have
been put on hydroxyurea (an anti-sickling agent) as a preventative
measure, but there is still a 1 7% recurrence rate in children.

112
IIAEMATOLOGY ANSWERS

Case 3
1 D Iron deficiency anaemia
The indices show a microcytic hypochromic anaemia. The MCV is
reduced in proportion to the degree of anaemia. In lhalassaemia trait, the
MCV is disproportionately reduced lo Ihe degree of anaemia (eg I lb 9.5
g/dL, MCV 50 l"L). The rod cell count is increased in thalassaemia trail
due to ineffective erylhropoiesis and decreased in iron deficiency due to
reduced production. The RDW (red cell distribution width) is usually
increased in iron deficiency due to the many elliptocytes/pencil cells. A
Ihrombocytosis is commonly seen if iron deficiency is a result of chronic
blood loss. In this case it was secondary to monorrhagia.

2 I Microcytosis
F Hypochromia
B Pencil cells

Using the central lymphocyte as a guide, one can see that most of the red
cells in this film are smaller, thus it shows a microcytosis. Many are
hypochromic with the central pallor being more than one-third of the
diameter of the cell. There are elliptocytes/pencil cells. Target cells are
also commonly seen, but are not demonstrated in this film.

Case 4
1 C Chronic myeloid leukaemia
A white cell count of 700 x 10'VL in ALL, AMI or high-grade NHL is
incompatible with life. It is possible to have a white cell count of 700 x
10''/L in CML, CLL and, very rarely, follicular lymphoma. The presence
of splenomegaly can be seen in all three diagnoses, but is most common
in CML. The thrombocytosis is suggestive of a myeloproliferative disease
and is commonly seen at presentation in CML. In the other two
conditions the platelet count is more likely to be low because of a
production problem due to heavy bone marrow infiltration but it may be
normal.
The film shows many neutrophils and myeloid precursors
(promyelocytes, myelocytes and metamyelocytes and the occasional
blast). Three eosinophils and one basophil are also seen (pink and purple
granules in the cytoplasm respectively). Basophilia and eosinophilia are
a big clue to a myeloproliferative disease as there is proliferation of all
myeloid lines. A count this high, and with an eosinophilia and basophilia
could not be confused with a reactive picture. A neutrophil alkaline
phosphatase (NAP) score can be used to differentiate a reactive picture
from an early CML. In the latter it is low and in the former it is raised.

1 13
HAEMATOLOGY - ANSWERS

Priapism in CML is due to leucostasis in the corpora cavernosa.

2 D t(9;22)
In 95% of cases of CML there is a reciprocal translocation between
chromosomes 9 and 22. The Philadelphia chromosome is a chromosome
22, from which the long arm is deleted (see diagram). This results in
production of a chimeric gene called BCR-ABL which defines CML.

95% of all cases of CML are Ph+

Case 5
1 B Fungal infection (typically Aspergillus)
The patient is pancytopenic as a result of chemotherapy given prior to
the administration of his sibling's pluripotent stem cells. The
conditioning chemotherapy ablates the patient's own marrow, removing
any residual leukaemia, and makes room for the donor pluripotent stem
cells; these engraft in the patient's bone marrow and divide to repopulate
the marrow and produce the patient's white cells, red cells and platelets.
During this phase the patient is neutropenic for 2-4 weeks. At this time
they remain isolated, often in rooms with high-efficiency particulate air
(HEPA) filtration to reduce infection exposure and are on prophylactic
antibacterials (ciprofloxacin), antivirals (aciclovir) and antifungals
(fluconazole or itraconazole).

114
I IALMA rOLOGY ANSWERS

The prolonged neutropenia puts bone marrow transplant patients at risk


of fungal infections, which can be ubiquitous. This presentation of high
fevers, pleuritic chest pain and haemoptysis and a CXR showing a
circular opacity in the right upper lobe is entirely consistent with
Aspergillus. The other diagnoses that should be considered are a
Staphylococcus aureus abscess, hospital-acquired pneumonia and
tuberculosis (check BCG scar/ethnicity/past history). The CXR is not
typical of I'CP, which usually shows bilateral ha/y consolidation,
maximal in the mid- and lower zones. The CXR in CMV or RSV
pneumonitis tends to show bilateral diffuse interstitial changes.
Hospital-acquired pneumonias tend to be Gram-negative Klebsiella,
Pseudomonasand coliforms can be seen. In immunocompromised
patients there is often focal consolidation. TB is usually secondary to
reactivation of old disease so look for old calcification in the upper
zones. If there is a past history of TB, the patient will be put on isoniazid
prophylaxis at the time of the transplant until count recovery.
The patient was started on broad-spectrum antibiotics (eg ceftazidime or
tazobactam and gentamicin) because a Cram-negative infection cannot
be excluded, but with the above history and radiological findings an
antifungal should be commenced. This would be amphotericin B, the
liposomal formulation if there is any evidence of renal impairment. This
would be continued for at least 6 weeks. The diagnosis is confirmed by
high-resolution CT. There are specific, radiological leatures which are
suggestive of a fungal infection nodules and spiculated lesions as seen
in the CT, a halo sign due to surrounding oedema and a crescent sign
due to cavitation within the fungal lesion. Very rarely, the Aspergillus
species can be isolated on bronchoscopy. Sensitivities can be carried out
if the fungal species can be isolated. Definitive diagnosis is usually by
CT-guided biopsy if feasible. The platelet count should be at least 50 x
1()''/L for this procedure.

Case 6
1 F Haemolysis
The bloods show a reticulocytosis, a raised unconjugated bilirubin and
LDI I and a strongly positive DCT with IgG and complement. The film
shows polychromasia and spherocytes. Hypersplenism will also be
contributing to the anaemia by red cell pooling. This will be causing the
mild leucopenia and thrombocytopenia. The iron studies - low/normal
iron, low TIBC and low transferrin saturations - are consistent with the
anaemia of chronic disease which will also be contributing. A
production problem due to heavy bone marrow infiltration is unlikely
due to the reticulocytosis.
HAEMATOLOCY - ANSWERS

2 A Lymphoproliferative disorder
B Chronic liver disease (with portal hypertension)
G Myelofibrosis
The above three diagnoses could cause this degree of splenomegaly and
hypersplenism. In chronic liver disease one should expect to see
peripheral stigmata of disease on examination. Autoimmune haemolysis
alone would not give this degree of splenomegaly. The history is too long
for acute myeloid leukaemia and this degree of splenomegaly would be
unusual. Autoimmune haemolysis is most likely to be associated with a
lymphoproliferative disorder as is the paraprotein.
NB After infective causes of hepatosplenomegaly had been excluded in
this gentleman, he was diagnosed with a rare T-lymphoproliferative
disorder which is associated with rheumatoid arthritis, by demonstrating
a T-cell clone by polymerase chain reaction (PCR) in the bone marrow
and splenic tissue after splenectomy.

Case 7
1 I Thymoma
K Thyroid tumour
E Teratoma
A Hodgkin's disease
B Mediastinal non-Hodgkin's lymphoma

The CXR shows an anterior mediastinal mass. This is suggested by the


presence of the silhouette of the descending aorta.

116
HAEMATOLOGY - ANSWERS

A CT chest will delineate the anatomy and associations of the mass and
demonstrate other lymphadenopathy. An abdominopelvic CT scan
should also be performed, looking for significant lymphadenopathy
elsewhere; if it were to be present the diagnosis would almost certainly
be lymphoma and the scan will stage the disease. II Ihere is no
better/safer target to biopsy, the mediastinal mass will have to be
approached via mediastinoscopy.
All these patients should be assessed for superior vena cava obstruction,
the symptoms of which are dyspnoea, orthopnoea, paroxysmal nocturnal
dyspnoea, facial and neck swelling, headache, and cough; the signs are
facial oedema, neck vein distension, tachypnoea and collaterals on the
thorax.

2 D Red cell aplasia


She has a normocytic anaemia with normal haematinics and a
reticulocytopenia. This suggests a production problem. The absence of
reticulocytes is consistent with a diagnosis of acquired red cell aplasia.
Parvovirus infection can cause a transient red cell aplasia but this is only
significant in patients with a shortened red cell survival (as in haemolytic
anaemias) where it can produce an acute life-threatening anaemia. It
could be an immune-mediated red cell aplasia, either antibody or T cell-
mediated or in association with neoplasia (thymoma, lymphoma or
carcinoma).

Red cell aplasia and a mediastinal mass is suggestive of a thymoma or


lymphoma.

3 F Tensilon® test
Double vision on reading (as the eyes tire) suggests fatigue, which is the
hallmark of myasthenia gravis and in the context of a mediastinal mass
and pure red cell aplasia this is suggestive of thymoma-associated
myasthenia gravis. The thymoma can be benign or malignant. A
Tensilon® (intravenous edrophonium) test is usually only necessary in
cases where there is some diagnostic, difficulty tie negative anti-ACh
receptor antibodies and an unhelpful EMG).

Case 8
1 G Myelofibrosis
H Tropical splenomegaly
E Leishmaniasis

This is massive splenomegaly: the causes are CML, myelofibrosis,

117
HAEMATOLOGY - ANSWFRS

malaria (and tropical splenomegaly in areas where malaria is endemic),


visceral leishmaniasis and splenic lymphoma. CML is excluded as the
WCC is entirely normal and the anaemia has been present for 3 years. In
splenic lymphoma the splenomegaly may be associated with
lymphadenopathy and/or circulating abnormal lymphocytes or it may be
an isolated phenomenon. If the latter is the case the diagnosis may have
to be made following splenectomy.

2 A Bone marrow
F Malarial antibody titres
D Leishmania serology
A bone marrow is diagnostic in myelofibrosis, but in many
circumstances it is impossible to aspirate because of extensive fibrosis; a
core of bone marrow called a 'trephine' is therefore necessary to make
the diagnosis. A peripheral blood film can be highly suggestive (see
below).
Tropical splenomegaly is suggested by malarial antibody titres. A
polyclonal elevation of serum IgM is also commonly seen.
In leishmaniasis Ciemsa-stained bone marrow smears may be diagnostic.
The aspirate can also be cultured. Percutaneous splenic aspiration for
staining and culture also has a high diagnostic yield but is dangerous if
there is thrombocytopenia or coagulopathy. Serological tests for
leishmaniasis can also be helpful.

3 A Tear-drop poikilocytes
I Leucoerythroblastic blood film
The diagnosis is myelofibrosis. The film shows anisocytosis and
poikilocytosis (variation in size and shape of the red cells respectively).
In particular, tear-drop poikilocytes are clearly seen. These occur in bone
marrow fibrosis or severe dyserythropoiesis. Myelofibrosis is a
myeloproliferative disorder. Early in the course of the disease there may
be a leucocytosis and thrombocytosis. Later there may be a
pancytopenia due to production failure as a result of extensive bone
marrow fibrosis. The splenomegaly, which can be massive, is a result of
extramedullary haemopoiesis. The film also shows a nucleated red cell
in the bottom right-hand corner and a myelocyte, which is a myeloid
precursor, in the top right-hand corner. It is therefore referred to as a
'leucoerythroblastic blood' film. These precursors are present in the
peripheral blood again because of extensive bone marrow fibrosis.

118
HAEMATOLOGY - ANSWERS

Case 9
1 F Sarcoidosis
The combination of bilateral hilar lymphadenopathy, a papular rash,
lung disease with non-casealing granulomas on biopsy, and a
lymphocytosis on BAL are highly suggestive of sarcoidosis. Acute
sarcoidosis can present with a symmetrical polyarthropathy. The main
differential diagnoses are tuberculosis and lymphoma; both of these
diseases can cause non-caseating granulomas and bilateral hilar
lymphadenopathy. The diagnosis of sarcoidosis is favoured by the
multisystem presentation with skin and joint involvement. The lack of a
fever goes against tuberculosis.

2 C Secondary polycythaemia
As a result of chronic hypoxia, over time this gentleman has developed a
secondary polycythaemia. His haemoglobin at presentation was 1 5.3
g/dl and now has increased to 1 8.2 g/gL as a result of hypoxia driving
bone marrow erythropoiesis. He went on to have a radio-labelled red
cell mass study which confirmed a true polycythaemia. An apparent
polycythaemia occurs when the red cell mass is in the normal range but
the plasma volume is decreased, giving the appearance of a
polycythaemia (for example in dehydration). The treatment of secondary
polycythaemia is dependent on symptoms such as headaches and
dizziness. Venesection is performed if there are symptoms or if the
haematocrit is repeatedly more than 0.5.5. Venesection would aim to
bring the haematocrit into the normal range (<0.52 in men and <0.47
in women). Studies have shown that a reduction in the haematocrit leads
to an increase in exercise tolerance. Oxygen therapy is also effective and
improves prognosis.

Case 10
1 G Acute myeloid leukaemia
E Myelodysplasia
H Hairy cell leukaemia
The blood results show a pancytopenia with the platelets and white cells
being most severely affected. The anaemia is normocytic and there is a
reticulocytopenia (absolute count reduced, % lower limit of normal
because of the reduction in the total red cell count) suggesting a
production problem. The three diagnoses in the list that can cause this
degree of pancytopenia are:
• Acute myeloid leukaemia - the normal bone marrow is replaced

1 19
HAEMATOLOCY ANSWERS

almost completely with leukaemic blasts which eventually spill out


into the peripheral blood, leading to a raised white cell count, if no
treatment is given.
• Myelodysplasia - an acquired bone marrow failure disorder affecting
all three cell lines, commoner in the elderly. Genetic defects occur
within multipotent stem cells, which lead to ineffective haemopoiesis
due to excess apoptosis and therefore peripheral cytopenias. It is a
pre-leukaemic condition, although death may occur prior to
transformation to AML. This condition is often but not always
associated with a macrocytic anaemia.
• Hairy cell leukaemia - this is characterised by a pancytopenia. This is
a result of reduced bone marrow production due to infiltration by the
leukaemia and hypersplenism. Splenomegaly is seen in 60% of cases
and can be moderate. Patients are commonly neutropenic but a big
clue to the diagnosis is a monocytopenia. Circulating hairy cells can
sometimes be seen on the peripheral film (B lymphocytes with an
irregular or hairy cytoplasmic outline).

2 A Acute myeloid leukaemia


F Malaria falciparum
The first picture shows falciparum ring forms (delicate rings with double
dots) within the red cells to the right of the centre of the field and in the
bottom right-hand corner of the field. The second picture shows
leukaemic blasts in a circle with one myelocyte at 1 o'clock. The blasts
are large cells with very little cytoplasm, with an open, lacy chromatin
pattern and some of them have clear nucleoli.

Chronic lymphocytic leukaemia and non-Hodgkin's lymphoma are


excluded as there is no excess of lymphocytes in either field. No hairy
cells are seen, the appearance of which is described above.

Case 11
1 E Malaria film
G Blood cultures
A Group and save
I le has just returned from a malaria region and is pyrexial. Malaria has a
higher mortality in patients with homozygous sickle cell disease than in
normal individuals.
Blood cultures should be done in all pyrexial patients with sickle cell
disease prior to commencing antibiotics because of the increased risk of
septicaemia with encapsulated bacteria and increasing resistance to
antimicrobials.

120
HAEMATOLOGY - ANSWERS

A group and save should be done immediately because he obviously has


phenotypically severe sickle cell disease and has required ITU admission
on two occasions. The blood transfusion laboratory should also be
informedof his admission if he is likely to require a blood transfusion, as
he will require sickle-negative blood which is specially matched to his
red cell antigens to prevent antibody formation. This type of blood may
need to be ordered from the blood service.
The macrocytosis is due to the hydroxycarbamide (hydroxyurea) therapy,
not B ij or folate deficiency. It can be used to monitor compliance with
the drug. Hydroxycarbamide is an anti-sickling agent. One of its modes
of action is to increase the ItbF %, which has an anti-sickling affect.

fie will require a chest X-ray, but as his chest is clear and his saturations
are maintained this does not need to be done immediately.
HbS % measurements are done following an exchange transfusion as the
aim is for the HbS % to be less than 30% in an adult to prevent further
sickle complications.

2 H None of the above


The film shows a number of target cells and some boat-shaped cells
(pointed at both ends). No classical crescent-shaped cells are seen in this
field. The number of sickle- or boat-shaped cells does not indicate the
severity of a sickle crisis. In the centre there is a red cell containing a
malaria ring form. This was found to be Plasmodium falciparum. He was
treated with quinine.

3 F G6PD levels
Prior to commencing quinine, G6PD levels should be checked in all
sickle cell patients. Co-inheritanceof G6PD deficiency is relatively
common as there is an overlap in the geographical distributions of the
two diseases, as in the heterozygous state both can protect against
malaria. As quinine can cause haemolysis in G6PD-deficient individuals
they should be closely observed following commencement.

Case 12
1 D ATLL secondary to HTLV-1 infection
HTI.V-1 infection is endemic in Japan, the Caribbean, Central and South
America and Central and West Africa. 2-4% of carriers develop ATLL (a
high-grade T-cel I leukaemia/lymphoma) over a 70-year life span.
Transmission is transplacental ly (rare), via breast milk, sexually or via

121
HAFMATOLOGY- ANSWERS

blood products containing! cells. Lymphadenopalhy,


hcpatosplenomegaly, skin lesions and hypercalcaomia are commonly
seen. Hypercalcaemia is due to the transcription of PTHrP (parathyroid
hormone related peptide) under the influence of HTLV-1 . In the
peripheral blood a lymphocytosis and the classic cells with
flower-shaped nuclei can be seen.
Sarcoidosis should be considered, but the lymphocytosis and normal
CXR make ATLL the most likely diagnosis.

2 F Strongyloidcs infection
Opportunistic infections, particularly Strongyloides, arc associated with
HTLV-1 infection because of defective T-cell immunity. All patients with
this diagnosis should be screened for Strongyloides. If present, they
should be treated with tiabendazole (thiabendazole) and this should
continue prophylactically throughout chemotherapy - usually CHOI'
(cyclophosphamide, doxorubicin hydrochloride, vincristine and
prednisolone). Prognosis is very poor, with virtually no long-term
survivors in ATLL.
In ATLL a lymph node biopsy would be indistinguishable from any other
high-grade T-cell non-Hodgkin's lymphoma. The diagnosis of ATLL rests
with HTLV-1 serology, which has to be positive.
Central nervous system (CNS) involvement can be seen in all high-grade
lymphomas, but is particularly common in ATLL. More than 10% of
cases have CNS involvement at post-mortem after relapse. All patients
therefore have intrathecal methotrexate with each intravenous course of
chemotherapy.
Case 13
1 D CML in blast crisis
An autologous bone marrow transplant describes a process where stem
cells are harvested from the bone marrow of the patient after
chemotherapy has been given to remove leukaemic or lymphomatous
contamination. These are then frozen. Then the patient is given ablative
chemotherapy and these stem cells are infused back into the patient to
rescue them from the ablative chemotherapy. They then engraft in the
bone marrow and divide to repopulatc the bone marrow and produce
the peripheral cells. This is now known not to be a curative treatment in
the vast majority of patients with chronic leukaemias and acute myeloid
leukaemias, but can prolong survival if there is not an option of a sibling
or unrelated transplant.
Although there was no evidence of the Philadelphia chromosome

122
HALMATOLOCY ANSWERS

(t(9;22)) on cytogenetic analysis for those 1 9 years, there would have


been copies of the BCR-ABL transcripts by I'CR (see Case 4}.

The bloods show a pancytopenia with profound thrombocytopenia and


neutropenia. This is not suggestive of chronic myeloid leukaemia in
chronic phase as there would be a raised white cell count and
thrombocytosis. The bone marrow is packed with a monomorphic
population of large cells with minimal cytoplasm, some of which show
vacuoles within the cytoplasm. This therefore has to be acute leukaemia
or blastic transformation of ((ML. The reappearance of the Philadelphia
chromosome on cytogenetic testing suggests this is blastic transformation
of CMI .

2 A B lymphoid
The T-cell markers are negative (CD2/3/7). The myeloid markers (CD 1 3
and 33) are also negative but the B-lymphoid markers are positive
(CD 19/10/79a).

This is therefore a lymphoid blast crisis of CML which is seen in a third of


cases of ((ML that transform. The remaining two-thirds show a myeloid
blast crisis.
Case 14
1 G Blood film
B Lupus anticoagulant and anticardiolipin antibodies
A Factor VIII levels

2 A Past obstetric history

3 D Antiphospholipid syndrome (APS)


The bloods show an isolated thrombocytopenia, (which should be
confirmed on a film) and a prolonged APTT, which when normal plasma
is added in a 50:50 mix does not correct. This is suggestive of an
inhibitor being present rather than a factor deficiency (if there was, for
example, a factor VIII, IX, XI or XII deficiency, which can cause an
isolated prolonged APTT, the addition of normal plasma containing the
factor VIII, IX, XI or XII would correct the APTT).
The inhibitor present could either be an acquired factor VIII inhibitor,
which can be seen in pregnancy, or a non-specific inhibitor such as a
lupus anticoagulant which causes a prolongation of the APTT in vitro but
is associated with a thrombotic tendency in vivo. An increased risk of
arterial or venous thrombosis, IUGR, recurrent miscarriage, premature

123
IAEMATOI OCY - ANSWERS

birlh with prc-eclampsia, and thrombocytopenia are all features of the


antiphospholipid syndrome. Thrombocytopenia and previous thrombosis
are not features of acquired haemophilia.
Testing for a lupus anticoagulant is by a clotting-based assay, usually the
dilute Russell viper venom time (DRVVT). IgM or IgC anticardiolipin
antibodies (aCL) are tested for using an ELISA. Either or both can be
present in the antiphospholipid syndrome. Eor the diagnosis to be made
there must be one clinical (vascular thrombosis or pregnancy
complications) and one laboratory feature - one or more of aCL or lupus
anticoagulant must be present on two or more occasions at least 6 weeks
apart.
Factor VIII levels can be measured to exclude an acquired factor VIII
inhibitor: in the presence of such an inhibitor the factor VIII levels would
be low.

Case 15
1 A Thrombotic thrombocytopenic purpura (TTP)
The bloods show evidence of haemolysis (reticulocytosis, raised bilirubin
and LDH) and there is a profound thrombocytopenia. The renal function
and clotting are normal. The film shows a number of red cell fragments -
most notably the fragment right in the centre of the field.

TTP consists of the pentad of fever, thrombocytopenia, microangiopathic


haemolytic anaemia (MAHA), neurological abnormalities and renal
involvement. The fragmentation is secondary to the microangiopathic
haemolysis. Many patients do not have all five criteria but institution of
therapy may be indicated. Neurological abnormalities range from
headaches to convulsions to a coma.
The other conditions that can cause a MAHA picture include HUS (but
the renal function is entirely normal) and HELLP syndrome- haemolysis,
elevated liver function tests and low platelets but the liver function is
entirely normal. Fever rarely occurs in HELLP and may be a useful
distinguishing feature.
Pre-eclampsia should be considered as proteinuria is demonstrated, but
the BP is normal. This degree of thrombocytopenia would be unusual
unless there was associated L)IC (the clotting and fibrinogen are normal).
There is an ongoing debate about whether TTP is precipitated by
pregnancy or whether the conditions occur together because pregnancy
and TTP occur most Irequently in the third and fourth decades of life.
However, relapses have been described in subsequent pregnancy.

124
HAEMATOLOCY - ANSWERS

2 B Plasma exchange
There is evidence from Canada that plasma exchange is superior to
plasma infusion. However, there is still a more than 10% mortality
associated with this condition and this is increased with delays in
commencing plasma exchange. TTP occurs because of production of an
autoantibody against the von Willebrand factor-cleaving protease
(vWF-CP), which normally cleaves unusually large vWF multimers
(ULvWF) into smaller functional multimers. The autoantibody leads to
destruction of the vWF-CP and accumulation of ULvWF multimers,
which leads to platelet aggregation and the formation of platelet
microthrombi, particularly within the brain and kidneys. Acquired TTP is
therefore another example of an autoimmune disorder.
Plasma exchange is the combination of plasmapheresis, which removes
Ul vWF multimers and autoantibodies against vWF-CP, and infusion of
plasma which contains vWF-CP. All patients should be counselled about
the risks of transfusion-transmitted infections prior to commencing
plasma exchange, as during the course of (he treatment they will have
extensive donor exposure.

As the occurrence of seizures is frequent in TTP (especially in children) it


is advisable to start anticonvulsants (phenytoin) prophylactically after
just one seizure.
Other supportive measures which should be instigated are a blood
transfusion if the patient is symptomatic, folic acid because of increased
requirements due to haemolysis, hepatitis B vaccination because of
blood product exposure, methylprednisolone 1 g daily for 3 days (if it is
not contraindicated because of the autoimmune nature of the disease),
aspirin when the platelets increase above 50 x 10'', with daily plasma
exchange, to reduce microvascular thrombosis.
Platelets are absolutely contraindicated in the absence of intracranial
bleeding or life-threatening haemorrhage as they will exacerbate
microvase uIar t hrombosis.

Case 16
1 .
A B i deficiency
G Myelodysplasia
The blood results show a macrocytic anaemia, a low platelet count and a
low-normal white cell count. The reticulocyte % and absolute count are
at the lower limit of normal but are lower than you would expect for this
degree of anaemia. The DCT is negative. The bilirubin and LDH are
elevated. The thyroid function is consistent with sick euthyroid or

125
HAEMATOIOCY- ANSWERS

secondary hypothyroidism (the former is common and the latter is rare).


The film shows a macrocytosis {compare the red cells with the
lymphocyte in the top of the field). Red cells should be roughly the same
size as a lymphocyte but many in this field are larger. Many are oval-
shaped (ovalocytes). The neutrophil in this field has a hypersegmented
nucleus (> 5 nuclear lobes). All of the above features are consistent with
a diagnosis of B,2 deficiency. It is rare to see this degree of macrocytosis
in any other condition. The raised bilirubin and LDH are due to
ineffective erythropoiesis within the bone marrow and early red cell
precursor destruction. The leucopenia and thrombocytopenia are usually
moderate, with the leucocyte count rarely being less than 1.5 x 10'VL
and the platelets rarely being less than 40 x 109/L. B]2 and folate
deficiency are indistinguishable on a blood film and indices.
If this was myelodysplasia the MCV would usually not be this high unless
there was co-existent liver or thyroid disease. The macrocyles would be
round rather than oval and the neutrophils would be dysplastic (reduced
granulation and hvpolobulated nuclei).
2 A B,2 and folate measurements

3 A B,2 replacement
D Digoxin
If Bi2 deficiency is confirmed there is no indication to do a bone marrow.
Regarding management blood transfusion should be avoided as a
cardiomyopathy is associated with B u deficiency and the administration
of blood can exacerbate any heart failure already present. If it is
essential, only 1-2 units of packed red cells should be given.
The administration of large doses of folate prior to correction of Bk>
deficiency, if there is co-existent deficiency, can unmask or exacerbate
the neurological sequelae of B|2 deficiency (the most worrying being
subacute combined degeneration of the cord).
When giving B12 injections, consider potassium supplements as there is a
danger of hypokalemia, which will exacerbate cardiac arrhythmias.
After the administration of B i2 the reticulocyte count will increase on
days 2-3 and reach a maximum on days 5-7. The neutrophil and platelet
count will correct over 7 days.
As there is cardiac compromise (tacchycardia and basal crepitations
suggestive of failure) the patient should be loaded with digoxin in order
to control the rate.

The most likely cause of B|2 deficiency in an elderly gentleman is


pernicious anaemia (PA) although poor diet should be considered. The

126
HAEMATOLOOY - ANSWERS

gastric parietal cell antibodies (sensitive but less specific) and intrinsic
factor antibodies (more specific but less sensitive) should be checked. A
Schilling test should also be performed, where oral radio-labelled B is,.
given followed by an intramuscular injection of non-radioactive Bp.
Urine is then collected over a 24-hour period. Bp must be absorbed for it
to appear in the urine. Normal urinary excretion of radio-labelled Bu is
> 1 0%, but if there is PA or malabsorption it will be <5%. The Schilling
test should then be repeated with intrinsic factor to confirm that there is
increased urinary excretion with its addition. Prior to a Schilling test the
patient must always be Bp-replete because Bp deficiency itself can affect
the bowel mucosa and cause malabsorption. In practice, the Schilling
test is often unavailable, and once hypochlorhydria (of any cause) is
excluded if a fasting plasma gastrin is elevated, then PA is confirmed (this
is not standard practice).

Case 17
1 B Direct Coombs' test
C LDH

2 B Immune haemolytic anaemia

3 E Antibiotic treatment
The blood results show a macrocytic anaemia with a reticulocytosis. The
film shows polychromasia, spherocytes and a circulating nucleated red
blood cell (top right-hand corner, with a dark circular nucleus and
purplish cytoplasm). The neutrophils show toxic granulation which is
secondary to the ongoing pneumonia.
The diagnosis is immune haemolytic anaemia secondary to high-dose
benzylpcnicillin, which was given to treat the pneumonia. The
mechanism of the haemolytic anaemia in this case is production of an
antibody which is directed against a drug-red cell membrane complex.
Haemolysis will stop rapidly on cessation of the drug.
The DCT will be positive in an immune haemolytic anaemia and the
LDH will be raised because of increased red cell destruction.
The other differential diagnosis would be an acute gastrointestinal bleed,
secondary to the ibuprofen. However you would not see a reticulocytosis
in an acute bleed because there would not have been time for a bone
marrow response. The polychromasia and spherocytes would also not be
present.
A delayed blood transfusion reaction could give you an identical picture,
but there was no recent history of a blood transfusion in this patient.

127
I IAEMATOLOGY ANSWERS

There would also have to have been another blood transfusion in the
past (or a pregnancy in a woman), following which he would have
developed an antibody to a red cell antigen which he has now been
re-exposed to. Delayed transfusion reactions usually occur 5- 10 days
following a blood transfusion but can occur up to 2 1 days afterwards.

Case 18
1 F Chronic lymphocytic leukaemia
D Autoimmune haemolytic anaemia
The bloods show a macrocytic anaemia due to the reticulocytosis with
an elevated white cell count. The films show a polychromasia, a
spherocytosis and a lymphocytosis. On high power the lymphocytes
have a round nucleus and virtually no cytoplasm and appear
monomorphic. The nuclear chromatin is condensed and said to have the
appearance of cracked mud. These lymphocytes are mature and in
isolation are difficult to distinguish from normal, non-clonal
lymphocytes. They can be distinguished from acute lymphoblastic
leukaemia blasts by their small size and condensed chromatin pattern.
Blasts in ALL would be larger and have a more open lacy chromatin
pattern.

Hodgkin's disease can be excluded as it is never seen in the peripheral


blood. Diffuse large B-cell lymphoma is rarely seen in the peripheral
blood and is usually a terminal event. The large lymphoma cells would
also not have this appearance. Sezary syndrome, a T-cell lymphoma, can
give a peripheral lymphocytosis (by definition the lymphocyte count
must be more than 15x10'VL). Sezary cells are also mature lymphocytes
but they have a convoluted or cerebriform nucleus which differentiates
them from CLL cells.
To confirm whether a lymphocytosis is clonal and indeed whether it is T
or B in origin, peripheral blood immunophenotyping is performed. This
tells us the markers that are on the surface of the lymphocytes which are
specific to T and B cells and in combination can tell us the type of
leukaemia or lymphoma.

2 B 1,4, 8 and 9
Oral chemotherapy
High-dose steroids
Folic acid
Blood transfusion
High-dose steroids are necessary to treat the AIHA. Oral chemotherapy
is needed to treat the underlying cause of the AIHA, which is CLL. Folic

128
HAEMATOLOGY - ANSWERS

acid is given because of increased requirements because of increased red


cell turnover. A blood transfusion is necessary because he is
symptomatic with worsening angina and it would take time for the
haemolytic process to be switched off by the steroids. There can be delay
in providing blood, as the autoantibody can mask alloantibodies, which
must be excluded before blood can be issued.

Case 19
1 D Parvovirus-inducer! red cell aplasia
This man's haemoglobin has dropped acutely in association with a viral
infection, which he has caught from his son. Ile has a reticulocytopenia,
so the drop is due to aplasia rather than acute haemolysis, where the
reticulocyte count would go up. There is a lymphocytosis and we are
told the film showed atypical (or reactive) lymphocytes, these are typical
of a viral infection. Parvovirus infection causes red cell aplasia. It is
tropic to erythroid precursors and infection leads to a virtual cessation of
erythropoiesis for 5 10 days. In any patient with a haemolytic anaemia
with shortened red cell survival this can cause a life-threatening
anaemia.

2 A Gilbert's syndrome
In hereditary spherocytosis the gallstones formed are pigment gallstones
as a result of the unconjugated hyperbilirubinaemia. In Gilbert's
syndrome, there is also an unconjugated hyperbilirubinaemia. Therefore
in patients with haemolytic anaemias who also co-inherit Gilbert's
syndrome, there will be an increased propensity to gallstone formation.

Case 20
1 C von Willebrand's disease
The abnormalities in the results given are a prolonged APTT and
bleeding time and a reduced factor VIII level.
A prolonged APTT can be due to:

• Factor VIII, IX, XI, or XII deficiency, ie intrinsic pathway factors


• von Willebrand's disease (vWD), due to a deficiency in Factor VIII (see
following page)
• A lupus anticoagulant - in vitro this prolongs the APTT but in vivo
leads to an increased thrombotic risk instead of bleeding
• Heparin therapy - this would also prolong the thrombin time, which is
more sensitive than the APTT to heparin.

129
HALMA TO I OGY ANSWERS

The bleeding time is prolonged in:


• Thrombocytopenia
• Uraemia
• Liver disease
• Chronic myeloproliferative disease
• Paraproteinaemia
• Anaemia
• Drugs - aspirin/alcohol/NSAIDs/p-blockers
• Inherited platelet function disorder/thrombocytopenia, eg
Glanzmann's thrombasthenia/Bernard-Soulicr syndrome
• von Willebrand's disease
• Significant haemophiliac disorders- A/B.
The mildly reduced factor VIII and the fact the patient is female suggests
that the patient is either a carrier of haemophilia A or has von
Willebrand's disease. Measurement of von VVillebrand factor (vWF) will
clarify this. As vWF acts as a protective carrier of factor VIII, a deficiency
leads to a reduced factor VIII level. vWF is also important in platelet
adhesion to vessel endothelium at sites of injury so a deficiency will
cause a prolonged bleeding time.
Von Willebrand's disease is a common genetic disorder affecting 1 2%
of the general population. It tends to present with easy bruising,
epistaxis, gastrointestinal bleeding and monorrhagia, ie mucocutaneous
bleeding.

130
NFUROLOCY - ANSWFRS

Chapter Two Answers

Case ÿ
1 D Wound botulism
There is currently an epidemic of wound botulism in injectingdrug
users. This is caused by the bacterium Clostridium botulinum, the spores
of which are present in soil. The organism grows in anaerobic conditions
such as necrotic injection sites in drug users - especially with the
practice of 'skin popping' (subcutaneous injection). The organism
produces botulinum toxin which inhibits pre-synaptic transmission in the
neuromuscular junction and in the autonomic nervous system. The
clinical presentation is normally with ophthalmoplegia and bulbar
dysfunction. Weakness then descends, spreading to involve the neck,
proximal limb muscles and respiratory muscles. There may be labile
blood pressure and abnormal pupillary reactions secondary to
autonomic involvement.
There arc occasional outbreaks of food botulism when the spores are
present in preserved foods. This normally produces marked
gastrointestinal features such as vomiting and diarrhoea as well as the
neurological sequelae.
If the patient had Guillain-Barre syndrome, one would expect more
sensory symptoms and signs.
Myasthenia gravis can produce a similar pattern of weakness. I lowever,
it does not produce autonomic dysfunction - in this case revealed by
unreactive pupils due to parasympathetic involvement.
Acute polio can produce an acute flaccid weakness, usually associated
with headache and meningism.
With an acute drug-induced myopathy, a higher CK would be expected.

2 D Get urgent anaesthetic review for transfer to ITU


The urgent need in this instance is for anaesthetic review and intensive
monitoring. One of the best measures of respiratory muscle function in
neuromuscular disorders is forced vital capacity. An FVC of less than 15
mL/kg (ie in a 70 kg man, 1 .05 litres) is certainly cause for concern. In
type II respiratory failure FVC is a belter marker for monitoring patients
than oxygen saturation. Arterial blood gases should also be performed to
look for CO2 retention. This patient also has a bovine cough (due to

131
NEUROLOGY- ANSWERS

impaired adduction of the vocal cords); he is at high risk of aspiration


and will need nasogastric feeding. Once the patient is stable,
infected/necrotic areas should be debrided, and systemic treatment given
with benzylpenicillin and botulinum antitoxin. Tissue and serum
samples should be sent to Microbiology for measurement of toxin,
culture ol organism and bioassay.
The table below shows some common features of neuromuscular
disorders and neuropathies which may present acutely. This is only a
guide and there may be atypical features.

Table 1 Common features of neuromuscular disorders and neuropathies

Guillain-Barre Miller-Fisher Botulism Myasthenia


syndrome variant of CBS gravis
Pattern of Ascending Descending Descending Proximal,
weakness fatiguable
Eye movement Sometimes Yes Yes Yes
disorder or
ophthalmoplegia
Autonomic Yes Some Yes No
involvement
Ataxia No Yes No No
Sensory signs Yes Yes No No
Reflexes Usually absent Usually absent Normal Normal
(sometimes
depressed)
CSF findings Elevated protein Elevated protein Normal Normal
Neurophysiology Slow conduction Slow CV Normal CV, small Decrement of
velocity (CV) motor action MAP on
potential (MAP) repetitive
stimulation
Serology Campylobacter Anti-ganglioside Clostridium Anti-
(20%), antibodies, in botulinum acetylcholine
anti-ganglioside particular receptor
antibodies anti-GQlb antibodies
(>90%) (70%)

132
NtU ROLOGY ANSWERS

Case 2
1 D Add dipyridamole and simvastatin
The history of this patient is consistent with a right hemispheric transient
ischaemic attack. He has vascular risk factors in that he is hypertensive
and a smoker. There have been a number of trials showing that carotid
endartereetomy should be considered if there is a greater than 60%
stenosis in a patient who has been symptomatic in the previous
6 months. In this case the stenosis is on the correct side in relation to the
patient's symptoms; however, it does not reach the threshold for surgery.
The risk-benefit ratio may be altered depending on the competency of
local vascular surgeons. A trial will report shortly on surgery for
asymptomatic carotid artery stenosis.
This patient has had a failure on aspirin and so further antiplatelet
therapy is warranted. It is very difficult to say which is the best option,
either replacing aspirin by clopidogrel or adding dipyridamole.' Aspirin
and dipyridamole has been shown to be more zeftective than aspirin
alone in stroke prevention. Trials in the use of clopidogrel are ongoing.
Given that the patient has carotid artery stenosis and borderline high
cholesterol, a statin should also be added. A recent study has shown that
40 mgof simvastatin reduces the risk of subsequent major vascular
events in high-risk patients, even if the total cholesterol is less than 5
mmol/L.2

' Antithrombotic Trial ists' Collaboration. 2002. Collaborative meta-analysis of


randomised trials of antiplatelet therapy for prevention of death, myocardial
infarction, and stroke in high-risk patients. BMJ, .524, 71 86.
2
Heart Protection Study Collaborative Group. 2002. MRC/BHF Heart Protection
Study of cholesterol-lowering with simvastatin in 20,536 high-risk individuals: a
randomised placebo-controlled trial. Lancet, 360, 7-22.

Case 3
1 A Sagittal sinus thrombosis
Ulcerative colitis results in thrombophilia and the biochemistry results
show that this patient is dehydrated. This puts the patient at risk of
venous sinus thrombosis. The CT head with contrast shows a
haemorrhagic infarct in the left frontal lobe with mass effect (this is a
venous infarct).

NB a unilateral intracerebral haemorrhage would not explain the clinical


signs on examination, (bilateral extensor plantars). The CT head also
shows what is called 'the empty delta sign'. An arrow points to the
sagittal sinus, which should be full of contrast. However, there is a dark
central area which represents thrombus within the sinus.

133
NEUROLOGY - ANSWERS

2 B Therapeutic anticoagulation with heparin


The patient should be therapeutically anticoagulated with heparin.
Although there is a haemorrhagic infarct, this is secondary to venous
hypertension. The consequences of not treating the thrombus are likely
to be dire. The patient clearly also needs fluid and electrolyte
resuscitation (5% dextrose is better for rehydration whereas normal
saline is more useful for volume replacement).

Case 4
1 C Right carotid artery dissection
On examination this patient has a right Horner's syndrome. He has
evidence of a right cortical deficit - sensory neglect is a cortical
phenomenon arising from lesions in the parietal lobe - he also has brisk
reflexes on the left. Carotid artery dissection often follows minor trauma.
It is more likely to occur in patients with a collagen disorder such as
Marfan's syndrome. Patients may complain of neck pain.

Regarding Horner's syndrome - the sympathetic outflow begins in the


hypothalamus and then passes through the brainstem via the lateral part
of the medulla. It exits via the T1 root and there is a synapse in the
superior cervical ganglion. There is a complex of sympathetic nerves
around the carotid artery and it is this which is damaged during carotid

134
NEUROLOGY ANSWERS

artery dissection. Sympathetic nerves then travel via the superior orbital
fissure into the eye. This patient has therefore had a right carotid
dissection; there is a high risk of thrombus formation at the site of
dissection and in this case thromboembolism has occurred into the right
middle cerebral artery territory.
Vertebral artery dissection normally results in a progressive brainstem
syndrome. Signs will often be bilateral and one would not expect
anterior circulation cortical signs such as sensory neglect.
Lateral medullary syndrome occurs following thrombosis of the posterior
inferior cerebellar artery. It can produce a Horner's syndrome; there are
usually associated ipsilateral cerebellar symptoms and signs, altered
facial sensation and IXth and Xth cranial nerve involvement.

2 B MRI brain and cervical spine


The best investigation is MRI brain and carotid arteries. The MRI cervical
spine gives views of Ihe carotid arteries and is required in particular to
look for thickening and thrombus within the wall of the carotid artery.
Magnetic resonance angiography may be performed in addition to show
narrowing of the artery. Percutaneous angiography is not normally
performed first-line due to its invasive nature.

3 C Therapeutic anticoagulation with heparin


The treatment of choice is therapeutic anticoagulation with heparin to
reduce the risk of further thromboembolism. He will subsequently need
treatment with warfarin.

Case 5
1 C Subarachnoid haemorrhage
The main differential diagnosis, clinically, is between meningitis and
subarachnoid haemorrhage. TheCT scan shows blood in the
subarachnoid space; it can clearly be seen outlining the midbrain and in
the Sylvian fissure (arrows). A large aneurysm can actually be seen in the
region of Ihe anterior communicating artery (arrowhead). In
subarachnoid haemorrhage meningism is common due to meningeal
irritation.

NB The patient does not have papilledema. Ilowever, in the context of


raised intracranial pressure, papilloedema can take up to 24 hours to
develop. On fundoscopy, subhyaloid haemorrhages (a haemorrhage
between the retina and vitreous) may be seen.

135
NEUROLOGY - ANSWERS

2 B Hydrocephalus
Hydrocephalus has developed as shown by the markedly dilated
temporal horns (asterisk). This patient needs urgent referral to the
neurosurgeons for treatment. In particular, given the presence of
hydrocephalus, not only does the aneurysm need treatment but
ventricular shunting may also be required. The patient should be treated
with nimodipine to prevent secondary vasospasm and kept well
hydrated. The options for aneurysm treatment include surgery to clip the
aneurysm or interventional radiology to coil the aneurysm. The choice
depends on the site and size of aneurysm and clinical condition of the
patient. The timing is also important as once vasospasm develops
(usually at day 3) it becomes a high-risk procedure.

Case 6
1 D Osteomalacia
Muscle disease often presents as a proximal weakness. Proximal
weakness in the lower limbs manifests itself as difficulty rising from
chairs and walking upstairs. People with proximal weakness in the upper
limbs may complain of difficulty hanging out the washing or combing
their hair.

136
NEUROLOGY - ANSWERS

Below are some causes of proximal weakness:


• Neuromuscular disorders:
myasthenia gravis
botulism
• Inflammatory myopathy:
polymyositis
dermatomyositis
• Infections of muscle:
staphylococcus
coxsackievirus
• Drug- induced myopathy:
statins
colchicine
corticosteroids
• Muscular dystrophy:
Duchenne muscular dystrophy
limb-girdle muscular dystrophy
• Metabolic and endocrine disorders:
hypoealcaemia secondary to hypoparathyroidism, vitamin D
deficiency
hypokalaemia
hypomagnesaemia
hypo- and hyperthyroidism
Cushing's syndrome
mitochondrial disorders.
Polymyalgia rheumatica is absent from this list, as although it can cause
muscle pain it should not cause weakness. Creatine kinase (CK) is a
useful marker of muscle disorders, where it is elevated. It is not elevated
in neuromuscular disorders such as myasthenia gravis. The other
conditions in this list can cause a raised CK and note that, particularly in
inflammatory myopathies, some drug-induced myopathies and muscular
dystrophies, it can be very high - in the thousands. Denervation of
muscle, as in motor neurone disease can also result in an increased CK.
In this case, the low calcium and elevated alkaline phosphatase (from
bone) point to osteomalacia, this is particularly common in Asians who
have moved to the UK, due to a combination of factors, including low
dietary intake and melanin in the skin decreasing vitamin Di formation.

137
NEUROLOGY - ANSWERS

Case 7
1 A Neurocysticercosis
The seizures he describes are consistent with simple partial motor
seizures followed by a Todd's paresis of the left upper limb. New-onset
partial seizures are always of concern as they indicate a possible
space-occupying lesion acting as an epileptogenic focus. Such patients
should therefore all be imaged. The CT scan shows multiple calcified
ring lesions of neurocysticercosis (arrows). This is due to CNS infection
with Taenia solium, a tapeworm whose life cycle involves pigs and
humans. It is most common in developing countries where humans
contract the disease by ingesting uncooked pork. Cysts have a
predilection for the brain but can also form in muscle, hence this
patient's thigh pain. As cysts degenerate they can produce an intense
inflammatory reaction. The most common presentation is with seizures;
they can also cause focal neurological deficits and hydrocephalus.
Definitive treatment is with albendazole in combination with
dexamethasone.

2 A Carbamazepine
Carbamazepine is an effective anti-epileptic for partial seizures. The
level is not yet therapeutic and so the dose should be increased. The
patient is stable and this is not an emergency situation, so loading with
intravenous phenytoin is not appropriate. Adding a second anti-epileptic
drug should be considered when the first is at target dose or if there are
dose-limiting side effects.

138
N It U R O L O G Y A N S VV t R S

Case 8
1 D Central pontine myelinolysis

Central pontine myelinolysis is a condition in which there is destruction


of myelinated tracts in the pons and cerebellum. It was first described in
alcoholics. It is caused if large and rapid shifts in plasma osmolality
occur, particularly during the correction of chronic hyponatremia. In
this case the patient had significant hyponatremia on admission -
secondary to vomiting and alcohol. As a guideline, hyponatremia
(especially if chronic) should not be corrected by more than 1 0 mmol/L
per day because of the risk of central pontine myelinolysis. Examination
findings usually reveal an ophthalmoplegia - due to involvement of the
Vlth cranial nerve nuclei and gaze centres within the pons as well as a
quadriplegia, which is often initially flaccid. There is no specific
treatment. A CT scan may be normal and an MRI brain which provides
better views of the brainstem and cerebellum is required (it may show
increased T2 signal within the pons). Wernicke's encephalopathy
secondary to thiamine deficiency can result in ophthalmoplegia, as well
as encephalopathy and depressed reflexes.

Case 9
1 D Paraneoplastic cerebellar degeneration
The key to this question is the history. This gentleman has a cerebellar
syndrome, which has progressed over a number of months. Ilis signs are
symmetrical, indicating bilateral cerebellar damage.
The normal MRI rules out space-occupying lesions such as a tumour and
also makes MS unlikely. In MS there are usually unmatched oligoclonal
bands present in CSF but not serum. This indicates intrathecal synthesis
and is useful diagnostically. Alcohol is likely to have a more insidious
course and both the history and blood tests do not point to excess
alcohol intake.
Friedreich's ataxia is a trinucleotide repeat disorder; it is inherited in an
autosomal recessive fashion. Other associated features include a
neuropathy, pes cavus, pyramidal signs in the lower limbs, optic atrophy,
scoliosis and cardiomyopathy. An earlier age of onset would be expected
and one may see cerebellar atrophy on imaging. I lerpes zoster
cerebellitis would have had an explosive onset.
Paraneoplastic cerebellar degenerations can be rapidly progressive.
Imaging may be normal in the early stage. The lumbar puncture may
often show a mildly elevated protein. The oligoclonal bands are matched
in CSF and serum, indicating a systemic immune response. The most

1 59
NEUROLOGY ANSWERS

likely primary in (his man's case would be small-cell lung cancer and he
needs a chest X-ray.
Lastly, severe hypothyroidism should not be forgotten as it may cause
cerebellar syndrome.

Case 10
1 C Neuralgic amyotrophy
Neuralgic amyotrophy is an inflammatory condition of the brachial
plexus in this case involving root level C5 and CO. The pathogenesis is
unclear but it often follows minor trauma or immunisation. The
presentation is very characteristic with severe pain which is then
followed by wasting and weakness in the relevant muscle groups, as well
as sensory and reflex impairment. Patients normally do make a degree of
recovery although this may take a number of months. This cannot be an
axillary nerve palsy as the findings are not restricted to the deltoid
muscle. Cervical root compression at multiple levels could produce a
similar picture but in this case the findings localise to C5 and C6, not C7.
Neoplastic infiltration of the brachial plexus is also a consideration.
However, this normally produces ongoing pain and is progressive. There
is no history of trauma to explain a brachial plexus avulsion injury.
Radiation can also cause a delayed brachial plexopathy (often many
years later) which is often painless.

Case 11
1 A Narcolepsy
The episodes she describes are a classic description of cataplexy. This is
a loss of muscle tone usually associated with an emotional experience
such as laughing, which normally lasts between a few seconds and
minutes. Narcolepsy is a condition characterised by excessive sleepiness
(which may be in inappropriate situations) and is strongly associated
with cataplexy. Other features of narcolepsy include sleep paralysis,
which occurs in the transition between sleep and waking and is the
transient inability to speak or move. Patients may also describe
hypnagogic hallucinations which may be very vivid and associated with
fear or dread at the onset of sleep. To investigate narcolepsy further a
sleep study can be performed: this shows a short sleep latency (of less
than 1 0 minutes) and early-onset RLM sleep. A baseline EEC will be
normal. There is a strong HLA association with HI A DR2. Treatment is
with central nervous system stimulants such as amphetamine-based
compounds and a newer non-amphetamine drug, modafinil. Cataplexy
responds to tricyclic antidepressants.

140
NEUROLOGY ANSWERS

Case 12
1 D Lambert-Eaton myasthenic syndrome
The patient has a fatiguable proximal weakness and this is the hallmark
of a neuromuscular disorder. He does not have anti-acetylcholine-
receptor (anti-ACh R) antibodies, though this does not definitely mean he
does not have myasthenia gravis as around 30% of patients with this
condition are anti-ACh R-negative. There are, however, some additional
features: he has clubbing and a normocytic anaemia, possibly indicating
an underlying malignancy. He also has some autonomic symptoms - dry
mouth and difficulty passing urine. These factors point to Lambert Eaton
myasthenic syndrome, which is a paraneoplastic condition strongly
associated with small-cell lung carcinoma. It is due to a defect in
pre-synaptic neuromuscular transmission due to anti-voltage-gated
calcium channel antibodies (these are diagnostic). Patients often initially
complain of proximal limb weakness and develop ptosis, diplopia and
bulbar problems at a later stage. There may also be associated
autonomic symptoms of blurred vision, dry mouth, constipation and
difficulty passing urine. Tendon reflexes are often depressed - unlike in
myasthenia. Electrophysiology is characteristic, with small motor action
potentials with a marked increase in amplitude following strong
voluntary contraction. These patients do not respond well to the
acetylcholinesterase inhibitors used in the treatment of myasthenia.
Symptomatic relief can be provided by 3,4 diaminopyridine; patients
may also improve when the underlying malignancy is treated.

Case 13
1 C Temporal lobe epilepsy
The patient symptoms are very characteristic of temporal lobe epilepsy.
Hallucinations are often experienced and these may be visual, auditory
or olfactory; a proportion of patients describe an unusual, rising
epigastric sensation. There may be strong emotional feelings of fear,
anxiety or happiness. The patient may describe deja vu or jamais vu.
These seizures are associated with an alteration in consciousness:
patients may feel disconnected from their surroundings and may be
described as less responsive by witnesses. There may be associated
automatisms during the seizure, such as lip-smacking, grimacing or
fumbling of the hands. If the seizure does not remain localised to the
temporal lobe, the patient may go on to have a secondary generalised
seizure. Febrile convulsions in childhood seem to predispose people to
the development of temporal lobe seizures and one theory is that this
leads to an alteration in the structure of the hippocampus called 'mesial
temporal sclerosis'. These changes can be seen with modern MRI.

141
NEUROLOGY - ANSWERS

The normal EEC (which is inter-ictal) does not mean that this is not
epilepsy. In a significant proportion of patients with epilepsy, a single
EEC will be normal and the critical diagnostic information is the history
(especially if there is a witnessed account) and one ought to repeat the
EEC.
Absences are usually brief episodes of loss of consciousness (lasting
seconds) which may be associated with flickering of the eyelids. The
patient's description of fear and anxiety may lead to confusion with
panic disorder but there are too many additional features here. Patients
may be more aware ol their heart beating; this does not mean they are
having an arrhythmia. All patients who present with loss of
consciousness should have an ECG. The association of palpitations and
anxiety does raise the possibility of phaeochromocytoma but this would
be unlikely to give rise to an alteration in consciousness.

2 C You advise her not to drive and that she should inform the DVI A
She is experiencing seizures and so she should not drive until she has
been seizure-free for I year. The exception to this is if she has only had
nocturnal seizures for at least 3 years. The fact that she does not lose
consciousness is irrelevant. You should inform the patient that she
should not drive and it is her responsibility to inform the DVI A. If one
were to do that oneself, as the patient's doctor, it would constitute a
breach of confidentiality.

Case 14
1 D Spinal abscess
The patient is pyrexial, he is an intravenous drug user and he has a
murmur: the underlying diagnosis is therefore infective endocarditis.
Staphylococcal organisms in particular have a high risk of metastatic
abscess formation, as in this case. Acute spinal cord syndromes can
present with a flaccid weakness and absent reflexes and have a
progressive onset. Plantar reflexes are helpful in this situation. The
sensory loss points to a level approximately up to the C5 dermatome.
The MRI cervical spine is a T2-weighted image (the CSF is bright). There
is a bright disc at C5/6, indicating discitis (arrow); the vertebral bodies
are also bright. The spinal cord is compressed at this level. Tuberculosis
of the spine (Pott's disease) can give similar imaging findings but a more
protracted course would be expected. An anterior spinal artery infarction
can occur in infective endocarditis due to thromboembolism; the
imaging findings and the involvement of the dorsal columns in this case
(which have a different blood supply) would not, however, be consistent
with this diagnosis.

142
NEUROLOGY - ANSWERS

2 D Broad-spectrum antibiotics and neurosurgical referral


This patient needs urgent neurosurgical referral for cervical cord
decompression. Antibiotics (including staphylococcal cover) should also
be given. It is also important to monitor respiratory function in such
patients - with serial vital capacity - as the phrenic nerve is supplied by
nerve roots C3, C4 and C5.

Case 15
1 E Right ulnar, left common peroneal

2 C Cryoglobulinaemia
This patient has mononeuritis multiplex (that is, a neuropathy picking off
individual nerves), affecting the left common peroneal and right ulnar
nerves.

143
NEUROLOGY - ANSWERS

The causes of mononeuritis multiplex are:


• Diabetes mellitus
• Systemic vasculitis/connective tissue disorders:
polyarteritis nodosa
Churg-Strauss syndrome
Wegener's granulomatosis
systemic lupus erythematosus
rheumatoid arthritis
Sjogren's syndrome
cryoglobulinaemia
• Infective:
HIV
Lyme disease
CMV
leprosy - depigmented patches?
• Sarcoid
• Paraneoplastic
• Malignant infiltration of nerves.
Chronic inflammatory demyelinating neuropathy can also cause an
asymmetric neuropathy. It is, however, often proximal and
electrophysiology will show evidence of demyelination - reduced
conduction velocity and/or conduction block.
There is a strong association between hepatitis C infection and the
development of mixed cryoglobulinaemia. Cryoglobulins are
immunoglobulins which reversibly precipitate in the cold (4°C). In this
case the rash, raised accelerated ESR, CRP and active urinary sediment,
in combination with mononeuritis multiplex, point to a systemic
vasculitic process and cryoglobulinaemia is the most likely cause. This
would need to be confirmed by taking blood at 37 °C and immediately
transferring it to the lab. Nerve conduction studies and FMG would also
be useful in this case to confirm mononeuritis multiplex.

Case 16
1 D Change the metoclopramide to domperidone
The patient has parkinsonism (the triad of rest tremor, rigidity and
bradykinesia). The patient's history is characteristic for idiopathic
Parkinson's disease. This usually has an asymmetric onset (in this case
the left arm), is gradually progressive and the symptoms are responsive to
levodopa. Patients with Parkinson's disease often deteriorate when
admitted to hospital. This may be because they are not receiving their
normal medication at the right time, or be due to intercurrent illness
(especially sepsis) or drug side effects. Drugs with antidopaminergic

144
NEUROLOGY - ANSWERS

activity, for instance neuroleptics such as haloperidol and anti-emetics


such as metoclopramide, should be avoided at all costs as they can
cause a marked deterioration. Domperidone is the anti-emetic of choice.
The 'Parkinson plus' syndromes demonstrate parkinsonism with
additional features. The onset is often symmetrical and they respond
poorly to levodopa therapy. In progressive supranuclear palsy there is a
characteristic eye movement disorder of reduced vertical gaze
(classically down-gaze, but up-gaze may also be affected); axial rigidity
is also a feature. Multiple systems atrophy (MSA) may be associated with
autonomic features such as postural hypotension and sphincteric
problems (MSA-A). MSA may also be associated with cerebellar features
(MSA-C). This patient was described as having a labile blood pressure
and it is tempting to think of MSA. However, this is more likely to be clue
to fluid balance problems related to recent surgery.

Case 17
1 C Ceftriaxone and ampicillin
The diagnosis is Listeria monocytogenes. This is a Gram-positive bacillus
which commonly affects patients at the extremes of age and the
immunosuppressed. Symptoms are usually of gradual onset. The
organism has a tropism for the brain substance as well as the meninges
(causing a meningoencephalitis) with particular involvement of the
brainstem (rhomboenccphalitis). Patients may therefore have reduced
conscious level, seizures and cranial nerve palsies. CSF findings typically
show a pleocytosis with a lymphocyte predominance; CSF glucose is
often normal. These findings are in contrast to other forms of bacterial
meningitis (most commonly Neisseria meningitidis, Streptococcus
pneumoniae and Haemophilus influenzae) where the CSF shows raised
polymorphs and a reduced glucose. In this instance Gram-positive bacilli
are seen in the CSF, but a positive Gram stain is only seen in 1 0-40% of
Listeria meningitis. Culture would be needed to confirm the diagnosis
and this should be performed on CSF and blood. The treatment of choice
is high-dose ampicillin.
Third-generation cephalosporins such as ceftriaxone are the treatment of
choice for conventional suspected bacterial meningitis. They should be
given as soon as the diagnosis is suspected.
Note that there has been a recent study showing that giving high-dose
dexamelhasone, starting with the first dose of antibiotic, in the treatment
of bacterial meningitis (and in particular Streptococcus pneumoniae)
improves prognosis. This may become accepted practice in the near
future.

145
NEUROLOGY- ANSWERS

Case 18
1 E Left midbrain
The photograph demonstrates that the left eye is deviated down and
outwards. In combination with the left ptosis, this indicates a left lllrd
cranial nerve palsy. The association with a right hemiparesis puts the
lesion in the left midbrain - Weber's syndrome. It is affecting the lllrd
nerve nucleus on the left as well as the corticospinal tract fibres before
they cross in the medulla, causing a right hemiparesis. It should be noted
that if the patient had a large space-occupying lesion in the cerebral
hemisphere it could cause transtentorial herniation of the temporal lobe
and therefore a lllrd nerve palsy associated with a hemiparesis.
However, the patient is clearly too well to be herniating.

With lllrd nerve palsies it is important to consider pupillary involvement.


The parasympathetic fibres travel on the outside of the lllrd nerve and so
are sensitive to extrinsic pressure. If the pupil is involved, (if the pupil is
dilated and unreactive) then there may be a lesion in the lllrd nerve
nucleus (as in this case), or there may be a mass lesion pressing on the
nerve. An important example to remember is a posterior communicating
artery aneurysm that can cause a painful pupil involving a lllrd nerve
palsy. If the pupil is spared, it is more likely that there has been a
microvascular event within the lllrd nerve, for instance a
mononeuropathy associated with diabetes.

Case 19
1 D Myasthenia gravis
The figure shows a significant decrement of the motor action potential on
repetitive stimulation (a decrement of greater than I5% is usually taken
as significant). This indicates a defect in neuromuscular transmission and
the most likely diagnosis is myasthenia gravis. The other finding one
would look for on single-fibre EMG is jitter. It should be noted that some
drugs, such as gentamicin, can also have effects on neuromuscular
transmission. On ITU this test obviously needs to be done when the
patient is not on any treatment with neuromuscular blocking agents. The
first-line investigation of myasthenia gravis is to test for
anli-acetylcholine receptor antibodies (positive in 70% of patients with
myasthenia gravis); in a case such as this it is not feasible to wait for this
result before doing neurophysiology. The Tensilon* test is now less
commonly used in the diagnosis of myasthenia gravis and this is now
reserved for difficult diagnostic cases (as when anti-ACh R antibody and
EMG are negative). In this test, the patient is pre-loaded with atropine,
edrophonium (a short-acting acetylcholinesterase inhibitor) is then given

1 4f>
NEUROLOGY ANSWERS

and the patient is closely observed for clinical improvement. Full


resuscitation equipment needs to be available as there is a risk of
bradyarrhythmia. Patients with myasthenia gravis also need a CXR in fhe
first instance (and usually CT chest) to look for thymoma.

Case 20
1 F Cavernous sinus thrombosis
The photograph shows bilateral proptosis and chemosis. An important
cause of cavernous sinus thrombosis is septic thrombophlebitis often
secondary to facial or sinus infections. The venous drainage of the eye is
into the cavernous sinus via superior and inferior ophthalmic veins and
hence thrombosis results in proptosis and chemosis secondary to raised
venous pressure. This is often associated with raised inlra-ocular
pressure. The cranial nerves III, IV and VI as well as the ophthalmic
division ol V lie within the cavernous sinus and therefore involvement
may cause a complete ophthalmoplegia and sensory loss in the
ophthalmic division of V.

2 D CT head with contrast


F Blood cultures
On a plain CT head the cavernous sinus may be seen as expanded and
hyperdcnse and on administration of contrast there is poor filling of the
cavernous sinus. MRI brain can provide excellent views of the cavernous
sinus but may be less available in an emergency situation. Certain MRI
sequences can be used to look specifically at the venous system. This
patient needs to be treated with broad-spectrum antibiotics, including
anaerobic cover (following blood cultures). Staphylococcus aureus is the
most common causative organism in septic cavernous sinus thrombosis.
The normal practice is to formally anticoagulate patients with heparin,
providing CT head shows no evidence of haemorrhage.
Ophthalmologists should be closely involved, particularly as intraocular
pressure may need to be reduced. This is a serious condition with a
mortality of up to 30%.

147
OPH IHALMOLOGY - ANSWERS

Chapter Three Answers

Case 1
1 A Left occipital cortex
By convention visual fields are recorded as what the patient sees.
Therefore the right eye is on the right and the left eye on the left. They
should be labelled to avoid confusion, but the position of the blind spots
tells one which eye is which. Most other recordings of a clinical
examination are recorded as if one is looking at the patient.
These fields show a homonymous hemianopia with macular sparing. The
calcarine cortex, at the occipital lobe receives its blood supply from the
two posterior cerebral arteries. The lip of the occipital pole, however,
receives a dual blood supply via the middle cerebral artery. It is at the tip
that macular vision is represented. Thus, with a stroke affecting the
posterior cerebral artery (here on the left side) a homonymous
hemianopia to all vision on the right occurs, but the patient will
fortunately be able to read as macula vision is spared. If the same process
were to occur on the right posterior cerebral artery the patient would be
left with the peculiarity of 'gun barrel' fields: the patient may not be able
to see a door, but can see a pin - this may arouse suspicion of hysteria in
the unwary physician.
A lesion in Meyer's loop the longer radiation that passes from the
lateral geniculate nucleus into the temporal pole before passing back to
the occipital cortex - affects the upper part of the field and thus the
patient would have a right homonymous upper quadrantanopia. Parietal
lobe lesions are the converse of this and affect vision below the level of
the eye, producing a left homonymous lower quadrantanopia.
Damage to the right optic nerve would cause unilateral blindness.

Case 2
1 B The findings are consistent with acute angle-closure glaucoma
H Acetazolamide 500 mg should be administered intravenously once
the diagnosis is confirmed
This is a classic story for acute angle-closure glaucoma. It is a disease of
middle to later life. An acute uniocular attack like this is often preceded
by a short period of blurred vision, often with halos round lights -
particularly at night-time. Pain is a variable feature - it can be so severe
and, along with the elevation in the intraocular pressure, cause nausea

148
OP HI I IALMOLOCY - ANSWERS

and vomiting such that the patient is assumed to harbour gastrointestinal


pathology, such as an obstruction. Acuity is reduced, the cornea appears
hazy due to oedema, there is often a ciliary flush, and the pupil is fixed,
often midpoint/dilated and ovoid. If the anterior chamber is viewed from
the side with a torch or a slit lamp, half the iris may be seen to be in
shadow and this suggests the iris is bulging forward due to the pressure in
the posterior chamber resulting from the obstructed flow of aqueous -
this is the shallow anterior chamber. A 'ciliary flush' refers to dilatation
of the deep conjunctival and episcleral vessels adjacent and
circumferential to the corneal limbus. It is best seen in natural light. It
suggests either anterior uveitis or glaucoma. If suspected, it requires
immediate assessment by an ophthalmologist as the optic nervecan be
permanently damaged by delay.
The pressure in the both eyes must be measured, and the elevated
pressure reduced. This is done with pilocarpine 2-4% drops hourly
(meiosis opens the blocked closed draining angle); acetazolamide is
given to slop the production of aqueous, usually orally, or inlravascularly
if they are vomiting (intramuscular injection is not favoured as it is
alkaline and often painful). Peripheral iridectomy (either by laser or
surgically) is performed once the pressures are normal - it is rarely
necessary as an emergency, if medical therapy fails to control the
pressure. Mydriatics must not be used as these will further increase the
block at the 'closed' angle.
Conjunctivitis does not cause perilimbic (within 3 mm of the cornea)
dilatation. It is usually itchy and is the only cause of a red eye that is so. It
is not truly painful and neither vision, reflexes or movements are
affected.
Anterior uveitis tends to cause a deep boring pain, of acute onset,
photophobia (because of iris spasm), blurred vision due to precipitates in
the aqueous, lacrimation, circumcorneal redness (ciliary flush), and a
small pupil (again because ol iris spasm) that may in time become
irregular or dilate irregularly due to adhesion formation. Pus may be seen
in the anterior chamber (hypopyon) with a slit lamp. It tends to affect
young or middle-aged patients, and is associated with arthr itides, such as
Still's disease and ankylosing spondylitis.

Case 3
1 B Diabetes me11 it us
This is a slide of diabetic pre-proliferative retinopathy. There are
extensive dot and blot haemorrhages; there are hard exudates, including
a circinate exudate at 1 1 o'clock to the macula; cotton-wool spots are
just seen, superior to the macula and superotemporal vascular arcade;

149
OPHTHAI.MOLOGY - ANSWERS

there is a large blot haemorrhage superiorly; the arteries are tortuous and
fine and the veins are irregular with beading. None of this would affect
acuity perse- in this case as evidenced by the satisfactory acuity with
pinhole correction; a loss of acuity would imply maculopathy. A
refractive error, or in this case lens opacity is reducing visual acuity,
which subsequently improves with pinhole correction, t his is difficult to
appreciate from direct fundoscopy but one can sometimes get a sense
that the physiological foveal pit reflex is lost, and the macula appears
boggy and oedematous. Fluorescein angiography can demonstrate
extensive capillary leak at the macula and show the true extent of the
vascular abnormalities, including intraretinal microaneurysms (another
pre-proliferative feature). Direct fundoscopy is not always entirely
satisfactory at detecting retinopathy. Fluorescein angiography is the gold
standard. Retinal photography should be performed for screening, and in
the UK this is undertaken by opticians and optometrists. In this case,
however, one should pick up the evident disease. The proximity of red
disease (dots and blots) to the macula is not worrying, and can be
followed up, but hard exudates (lipid-laden macrophages) and
cotton-wool spots (retinal ischaemia - which stimulates new vessel
formation) near the macula warrant a referral to an ophthalmologist,
particularly if extensive. Any loss of acuity -even if the fundus looks
normal - suggests maculopathy and warrants referral.
Ankylosing spondylitis is associated with anterior uveitis. Hypertriglycer-
idaemia is associated with the appearance of lipaemia retinalis where
the vessels look milky. Hypercholesterolaemia, whether combined or
solitary, can rarely be associated with retinal ischaemia, but only
because of proximal arterial occlusion. These lipids are in keeping with a
diabetic dyslipidaemia and the triglycerides are not high enough to cause
lipaemia retinalis. Multiple sclerosis causes optic neuritis. If behind the
eye, this could only be appreciated as scotomas or loss of acuity and
optic atrophy on fundoscopy. If at the eye, papillitis may occur where
the eye is painlui, with or without photophobia, and the disc looks
swollen and pink. Sarcoidosis can cause a retinal vasculitis or anterior
uveitis.

Case 4
1 A Blood pressure measurement
Ft Referral for intraocular pressure measurement
The slide shows a fundus with grade 4 hypertensive retinopathy with
optic disc oedema, cotton-wool spots and widespread A-V nipping. The
retina looks oedematous. The other features one may see are
flame-shaped retinal haemorrhages and hard exudate; which may collect
around the fovea producing a 'macula star'. The presence of end-organ

150
OPHTHALMOLOGY ANSWERS

damage like this dictates the urgency with which this condition is
managed. Obviously the blood pressure needs to be recorded and
optically controlled.
This requires urgent ophthalmological assistance if there is not to be
permanent damage to the optic nerve. Secondary optic atrophy with
permanent visual impairment can occur.
Considering the other consequences of hypertension, the following
ought to be done promptly: urinalysis, CXR and ECG ± echo to look for
renal damage and LVH/strain respectively. More broadly, one must
consider not only the consequences of the hypertension but also its
causes, such as renal or endocrine disease.
A CXR needs to be done but not at the expense of everything here. The
presence of a carotid bruit does not dictate the need to do a Doppler or
MRA carotids. This is not the appearance of choroidoretinitis, which may
be caused by Toxoplasma (which may make one think of HIV), or
dysthyroid eye disease. The latter can cause optic atrophy from optic
nerve compression if there is increased pressure in the orbit.
As a result of ischaemic proliferation of new vessels on the iris and in the
drainage angle (rubeosisiridis) may occur thus causing a secondary
glaucoma.

151
RHEUMATOLOGY ANSWERS

Chapter Four Answers

Case 1
1 A Wegener's granulomatosis
The presence of a multisystem disease with associated constitutional
symptoms should prompt the consideration of vasculitis as a primary
diagnosis.
Wegener's granulomatosis (WG) is a small-vessel granulomatous
vasculitis affecting any organ system. It most commonly affects the upper
and lower respiratory tracts and the kidney, with other systems including
the eye (retro-orbital pseudotumour), the skin, the musculoskeletal
system (arthralgia and myalgia more common than arthritis), the nervous
system (mononeuritis multiplex) and the heart (pericarditis).
ANCA (antibody to neutrophil cytoplasmic antigens) is not a definitive
test, but is often associated with vasculitis. WG is most commonly
associated with c-ANCA (cytoplasmic pattern of staining, as opposed to
perinuclear). The protein that the c-ANCA recognises is nearly always
protcinase-3 (PR3). PR3 has a sensitivity of -85% for WG, with a higher
specificity. It is, however, also associated with other vasculitides such as
microscopic polyangiitis. This diagnosis is less likely given the upper
respiratory tract involvement and the presence of the cavitating
pulmonary nodules.
Rheumatoid arthritis (RA) can cause mononeuritis multiplex and
cavitating pulmonary nodules. It is more commonly associated with
p-ANCA or atypical ANCA, when present. The diagnosis is unlikely,
however, given the absence of synovitis with no evidence of chronic
deformities.
Goodpasture's syndrome, like WG and systemic lupus erythematosus
(SLF), can affect the lung and kidneys. It is caused by circulating
anti-basement membrane antibodies. Binding of these antibodies leads
to activation of the complement system with subsequent inflammation,
causing glomerulonephritis and alveolar haemorrhage.

Causes of cavitation in the lung include:


• Carcinoma
• Autoimmune (rheumatoid arthritis)
• Vascular (pulmonary emboli, septic emboli)

152
RHEUMATOLOGY ANSWERS

• Infection (Staphylococcis aureus, Klebsiella, Pseudomonas,


tuberculosis, hydatid disease)
• Trauma.

Case 2
1 C Psoriatic arthritis
Radiological features of psoriatic arthritis can include a destructive
erosive arthritis, sometimes with a 'pencil in cup' appearance, periosteal
new bone formation, sausage-like swelling of the digits, terminal tuft
resorption and ankylosis of the distal and/or proximal interphalangeal
joints. The pattern of joint involvement can include the distal
interphalangeal joints. There is often a polyarticular unidigit pattern,
involving the metacarpophalangeal, and proximal and distal
interphalangeal joints of the same finger.
Whilst the majority of psoriatic arthritis patients will be negative for
rheumatoid factor, there is a small percentage that can have a low titre of
rheumatoid factor. It is important to remember that a positive rheumatoid
factor does not equate to a diagnosis of rheumatoid arthritis.

Case 3
1 B Coeliac disease
The raised calcium, low phosphate and raised alkaline phosphatase
generate the differential diagnosis of primary or tertiary
hyperparathyroidism. A diagnosis of primary hyperparathyroidism fails to
explain the low Bi2, folate and ferritin, however.
Tertiary hyperparathyroidism is most commonly seen in end-organ renal
disease and vitamin D deficiency. It can lead to X-ray bony changes such
as subperiosteal bone resorption, tufting of the terminal phalanges,
diffuse osteopenia, resorption of the distal clavicle and discrete lytic
lesions in bone (brown tumours) as seen on this X-ray. Other rheumatic
syndromes associated with hyperparathyroidism include painless
proximal muscle weakness, chondrocalcinosis, arthralgias (especially of
the proximal interphalangeal joints) and gout.
Whipple's disease tends to affect middle-aged white men, presenting
with a migratory oligoarthritis. They then gradually develop steatorrhoea,
diarrhoea and weight loss. Other features include pigmentation,
lymphadenopathy and neurological involvement, dementia being the
most frequent CNS symptom, jejunal biopsies show infiltration of the
lamina propria by large macrophages containing PAS+ve glycoprotein
granules.

153
RHLUMATOLOGY - ANSWERS

Enteropathic arthritis is an inflammatory arthritis associated with bowel


disorders, commonly inflammatory bowel disease. Both Crohn's- and
ulcerative colitis-associated arthritis predominantly affect the knee and
ankle. In this case, the knee pain is due to the lytic lesion rather than to
an arthritis. Enteropathic arthritides can also manifest as inflammatory
spinal arthritis similar to ankylosing spondylitis.

Case 4
1 B Limited cutaneous systemic sclerosis
Raynaud's phenomenon has many associations, including
immune-mediatedand occupation-related conditions, obstructive
vascular disease and metabolic disorders; it can be drug-induced or
related to infections. The strongly positive ANA in this case points
towards an immune-mediated condition, making hand-arm vibration
syndrome (formerly called 'vibration white finger') less likely.
The presence of dysphagia makes SLE a less likely diagnosis. The
remaining three can all have dysphagia as a presenting complaint. The
appearance of the Raynaud's 7 years before any other features of a
connective tissue disorder, in those that develop systemic sclerosis, is
more typical of the limited variety. The skin changes in diffuse cutaneous
systemic sclerosis tend to develop simultaneously with the Raynaud's.
The anti-centromere antibodies strongly support the diagnosis of limited
cutaneous systemic sclerosis.
The symptoms of mixed connective tissue disease are commonly hand
oedema, synovitis, myositis, Raynaud's and acrosclerosis/sclerodactyly,
with oesophageal dysmotility in two-thirds of patients. It is associated
with anti-U 1-RNP antibodies. Diffuse cutaneous systemic sclerosis
(DCSS) is more commonly associated with anti-topoisomerase 1 (Scl-70)
antibodies.
Patterns of nuclear staining have now been largely supplanted by
specific nuclear and cytoplasmic antigens. For reference, the
associations are as follows:
Homogenous or diffuse - Antibodies to DNA-histone complex
Peripheral or rim - Antibodies to DNA
Speckled - Antibodies to Sm (Smith), RNP, Ro, La, Scl-70 and others
Nucleolar - Antibodies to nucleolar RNA
Centromeric - Antibodies to centromeres

154
RHEUMATOLOGY ANSWERS

Case 5
1 E Severe osteoporosis
Osteoporosis is classified according to the T-score as follows:
• Normal T-score > -1
• Osteopenia T-score < - I, > -2.5
• Osteoporosis T-score < -2.5
• Severe osteoporosis T-score < -2.5 with fragility fracture
Osteomalacia is a metabolic bone disease resulting from impaired
mineralisation of mature bone, most commonly secondary to vitamin D
deficiency.
Bone mineral density (BMD) is commonly assessed by dual-energy X-ray
absorptiometry (DfXA). The results are expressed as an absolute bone
mineral density, recorded in g/cm?,a T-score and a Z-score. These two
'

scores are a comparison of the patient's BMD with that of reference


populations. The numerical T- and Z-scores are the number of standard
deviations away from the reference population mean.
The T-score represents the number of standard deviations the patient's
observed BMD is from that observed in normal young women. This
population is selected as it is the age group that has the maximum bone
mineral density, or 'peak bone mass'. Numerically, the T-score is
equivalent to:
Observed BMD -Peak BMD
Standard deviation of measurement
Therefore, a T-score of -2.0 means the patient is two standard deviations
below normal peak bone mass. The T-score determines whether or not
the patient has osteoporosis.
The Z-score represents the number of standard deviations the patient's
observed BMD is from that observed in an age-matched normal
population. The Z-score does not have any influence on the
classification of osteoporosis. It does, however, give an indication of
whether the patient's bone mineral density is appropriate for their age or
whether other factors may be responsible for an unusually low bone
mass.

Case 6
1 D Bisphosphonate
The T-score that triggers treatment is set at -1.5. Glucocorticoids
increase the risk of fracture over and above the effect of low bone

155
RHEUMATOLOGY ANSWERS

mineral density. Therefore, for a given bone mineral density, the risk of a
frac ture is higher in an individual on steroids than it is for an individual
on no treatment.
Bisphosphonates have data to support their use in steroid-induced
osteoporosis, but raloxifene (a selective oestrogen-receptor modulator)
does not.
Recombinant PTH (teriparatide, a daily subcutaneous injection for up to
18 months) is licensed for treatment of established osteoporosis in
post-menopausal women and is used in severe disease resistant to other
therapies.
Evidence-based guidelines for the management of glucocorticoid-
induced osteoporosis were published in 2002'.
!
http://www.rc piondon.ac.uk/pubs/books/glucocorticoid/glucocortCYinc ise.pdf

Case 7
1 E Gout
The characteristic X-ray changes with gout include bony erosions with
sclerotic margins, giving a 'punched-out' appearance. In contrast to
rheumatoid arthritis, the erosions are juxta-articular rather than
periarticular (away from the joint line rather than at the joint itself). This
gives the impression of an overhanging edge, as can be seen clearly at
the left middle PIP joint. In this case, the predisposing factor to
developing gout is the psoriasis: hyperuricaemia predisposes to gout.
There are many states of high turnover of nucleic acid that can lead to
hyperuricaemia. These include psoriasis, myeloproliferative and
Iy mphoproIiferat ive disorders.
The nodule palpable at the elbow represents a gouty tophus rather than a
rheumatoid nodule. Olecranon bursitis is a common finding in
individuals with chronic gout.
NB the presence of psoriasis and arthritis does not automatically mean
psoriatic arthritis.

Case 8
1 E Synovial fluid analysis of the right knee effusion
This patient has seropositive rheumatoid arthritis (RA) as evidenced by
the chronic deformities affecting the hands, wrists and feet, in addition to
the rheumatoid nodules. The history suggests that the disease was active
up to 20 years ago but then entered remission. The diagnosis is a clinical
one and therefore tests such as rheumatoid factor and hand and feet
X-rays are not necessary.

15b
RHEUMATOLOGY ANSWERS

The current problem is localised to her right knee. The differential


diagnosis would be a reactivation ol* her inflammatory arthritis, a crystal
arthropathy such as gout or pseudogout, or a septic arthritis. While she
may develop osteoarthritis in her knees, possibly secondary to her
inflammatory arthritis, this would not present with a hot, red, swollen
joint.
Of the above differential diagnoses, the most important one to exclude
is a septic arthritis, and this must always be considered when a
monoarthritis occurs. Although this may lead to an elevated white cell
count or positive blood cultures, neither are specific to the condition.
The only way to exclude a septic arthritis with confidence is to aspirate
the joint and examine the fluid by microscopy and Gram stain. Although
the prime reason for aspirating the joint as the first step is to exclude
septic arthritis, this fluid analysis will also be able to differentiate a
crystal arthritis, and between an inflammatory and non-inflammatory
arthritis.

Case 9
1 C Methotrexate
This patient has rheumatoid arthritis (RA). This is established from the
pattern of inflammatory joint involvement and supported by a strongly
positive rheumatoid factor. The X-rays are normal as is often the case in
early disease. Only 25% of patients in early arthritis clinics have erosions
on X-ray. Erosions can develop up to 5 years after disease onset.
This patient has many features of a potential poor prognosis (young male,
high titre of rheumatoid factor in early disease, large number of joints
involved, high ESR, large-joint involvement, significant disability).
Evidence shows that such patients should have early intervention with
disease-modifying antirheumatic drugs (DMARDS) in order to prevent
irreversible damage. This differs from the 'therapeutic pyramid' approach
of old where patients were initially treated with anti-inflammatory drugs
and only later with DMARDs.
The first-choice DMARD in early disease is debated. There are no
meta-analyses of therapy in early RA. However, methotrexate is better at
slowing disease progression than hydroxychloroquine in meta-analyses
performed in established disease.
There is currently insufficient evidence to support the use of biological
agents (TNF-a blockers) as first-line treatment. NICE guidelines state that
they may only be used when two conventional DMARDs have tailed,
and if the disease is sufficiently active as assessed by the DAS28 score.

157
RHEUMATOLOGY - ANSWERS

Case 10
1 D Disseminated gonococcal infection (DCI)
Arthritis complicates 1-3% of gonococcal infections. Two-thirds of
patients with DCI I present with tenosynovitis, fevers and dermatitis. The
rash may be maculopapular, vesicular or pustular. The extension of the
swelling over the dorsum of the hand in this case represents
tenosynovitis.
Associations between pustular skin lesions and rheumatic diseases
include inflammatory bowel disease, Behcet's syndrome and Sweet's
syndrome (acute febrile neutrophilic dermatosis).
The pustular lesions of Behcet's syndrome often occur at sites of minor
trauma, such as needlestick pathergy. In this case, there were no other
features of Behcet's syndrome.
Criteria for Behcet's syndrome are:
• Recurrent oral ulceration plus two of:
• Recurrent genital ulceration
• Anterior or posterior uveitis, cells in the vitreous by slit lamp
examination or retinal vasculitis
• Skin lesions:
erythema nodosum
pseudofol Iiculitis
papulopustular lesions or acneiform nodules in post adolescent
patients not on steroids
• Positive pathergy test at 24-48 hours.
The rash of Lyme disease is typically erythema chronicum migrans.
Arthritis is seen in 60% of infections, with about 1 0% of untreated
patients developing a chronic Lyme arthritis.
Acute rheumatic fever typically causes erythema marginatum, with the
joint involvement usually a fleeting, migratory polyarthritis affecting the
large joints (knees, ankles, elbows and wrists). The cutaneous lesions
characteristic of a reactive arthritis are keratoderma blennorrhagicum
and circinate balanitis, with joint involvement - typically asymmetric,
oligoarlicular(<5 joints) and involving the knees, ankles and/or feet.
Other well-recognised features are sacroiliitis and enthesitis
(inflammation at the insertion of tendons into bone).

158
RHEUMATOLOGY - ANSWERS

Case 13
1 E Polymyalgia rheumatica (PMR)
PMR rarely affects individuals under 60 years of age, and affects women
twice as often as men. The clinical features include stiffness and pain in
the shoulder and pelvic girdles, systemic features of debility, weight loss,
tiredness, a low-grade temperature and depression, and an accelerated
ESR (but not always).
There are no universally accepted diagnostic criteria, but one set requires
three or more of the following:
• Age >65
• ESR >40 mm/h
• Bilateral upper arm tenderness
• Morning stiffness of > 1 hour
• Onset of illness < 2 weeks
• Depression and/or weight loss.
RA remains a possibility, and can start with a polymyalgic presentation.
The knee effusion could be present in both conditions. Synovitis is well
described in PMR, usually affecting the knees, wrists or sternoclavicular
joints. The presence of symptoms consistent with temporal arteritis, or
giant cell arteritis (pain on chewing or jaw claudication) make PMR the
more likely diagnosis as there is a well-recognised association between
the two.
Although osteomalacia can cause pain and weakness in a similar
distribution, there is nothing further to support this diagnosis, other than
a marginally raised ALP. The calcium and phosphate arc both normal
(usually low) and there is no evidence of Looser's zones on the X-ray.
Polymyositis is unlikely given the normal creatine kinase.
Fibromyalgia is a disease of chronic widespread pain, where more than
11 out of 1 8 specific trigger points are tender. The absence of axial
tenderness in this case makes the diagnosis unlikely, and fibromyalgia
would fail to explain the high ESR and the synovitis.

Case 14
1 A Ankylosing spondylitis
The X-ray shows fusion of the sacro-iliac joints and vertical
syndesmophytes, typical of ankylosing spondylitis. Syndesmophytes
represent calcification of the outer layers of the annulus fibrosis. The
earliest lesions visible are Romanes lesions, or 'shiny corners', followed
by 'squaring' of the vertebral bodies. The pathological changes are an

160
RHEUMATOLOGY ANSWERS

initial enthesopathy (inflammation at the insertion of ligament into bone)


followed by calcification along the ligament. Once these
syndesmophytes meet from the vertebral body above and below an
intervertebral disc, a bony bridge forms; this gives the appearance of a
'bamboo' spine.
Nocturnal and morning spinal stiffness is characteristic of inflammatory
spinal disease. Decreased spinal mobility can be measured by the
modified Schober's lest. This measures the increase in distance between
two marked points along the lumbar spine, the upper 10 cm above and
the lower 5 cm below an imaginary line drawn between the dimples of
Venus. The increase on forward flexion should be more than 5 mm.
While an increased finger-to-floor distance may represent reduced spinal
flexion, the ability to touch the toes does not exclude reduced spinal
flexion as the manoeuvre can be achieved through good hip flexion
despite a fused lumbar spine.
Extraspinal features of ankylosing spondylitis include:
• Peripheral Arthritis (large joints commonly, plus sternoclavicular,
temporomandibular joints, cricoarytenoid)
• Aortic insufficiency
• Atlanto-axial subluxation with associated neurological sequelae
• Apical fibrosis
• Anterior uveitis
• IgA nephropathy
• Amyloidosis
• Osteoporosis.
Reiter's syndrome is classically described as the triad of arthritis,
conjunctivitis and urethritis. It may present with low back pain and a red
eye. The diagnosis is not this, however, as the radiological features of the
spine differ. In Reiter's, and in other seronegative spondyloarthropathies,
there can be large 'jug-handle' syndesmophytes, usually in a
non-symmetrical distribution. The osteophytes of spondylosis are
horizontal as opposed to the vertical syndesmophytes of ankylosing
spondylitis.

Case 15
1 C Reactive arthritis
The pain around his right foot and ankle is suggestive of an enthesitis
(inflammation at the insertion of a ligament or tendon into bone). The
pain in his sole, worse on weight-bearing and worse in the morning, is
typical of plantar fasciitis. The pain at the back of the ankle refers to
Achilles tendonitis. Enthesitis is commonly found in the seronegative

161
RHEUMATOLO G Y A NS W t R S

spondyloarthropathies, and is a frequent finding in reactive arthritis.


Other typical musculoskeletal features include arthritis, usually
oligoarticular and more commonly lower limb, and spondylitis.
Reactive arthritis occurs primarily in young individuals. It is a sterile
inflammatory synovitis following infections, typically enteric or
urogenital. The arthritis can come on around 1-4 weeks after the initial
infection. Common infectious agents include Chlamydia, Yersinia and
Campylobacter. While the arthritis is triggered by the infective process,
the organisms themselves are not found within the joint.
Synovial fluid analysis can differentiate different types of arthritis. In
inflammatory joint disease the cell count exceeds 1 000 cells/mm' and in
non-inflammatory arthropathies it is usually less than 1 000 cells/mm1.

Case 16
1 15 Diabetes mel Iitus
This patient presents with three separate conditions. Her shoulder
symptoms are typical of adhesive capsulitis, or 'frozen shoulder'. This
condition is usually seen in patients over 40, and it is more frequent in
diabetics.
Whilst the hands may resemble those changes seen in scleroderma,
systemic sclerosis would not explain the spinal features. The condition
shown is diabetic cheiroarthropathy, or diabetic hand syndrome of
limited joint mobility. It may be seen in up to 30 -50% of long-term
diabetics.
The spinal X-ray shows the diffuse idiopathic skeletal hyperostosis
(DISH), also known as Forestier's disease or ankylosing hyperostosis. It is
characterised by flowing calcifications along, but separated from, the
anterior border of the vertebral bodies. It involves at least four adjacent
vertebrae and is more commonly seen in the thoracic spine. It is also
associated with diabetes, particularly type 2. Involvement of the cervical
spine, as in this case, can present with dysphagia.

Case 17
1 E Osteoarthritis (OA)
The muscle weakness in this case may make you think of a primary
muscle disease such as polymyositis or dermatomyositis. However, these
would not cause the firm swellings described in the question. The
changes in the hand seen in dermatomyositis are Gottron's papules, an
erythematous scaling rash typically over the MCPs, PIPs, DIPs and
dorsum of the hand.

162
RHEUMAT OLOGY ANSWERS

Rheumatoid arthritis does not have DIP joint involvement, making this
diagnosis unlikely. The absence of any rash or nail changes make
psoriatic arthritis equally unlikely.
OA tends to present with pain in the area of the involved joints. This is
worse on activity, which differs from inflammatory arthropathies whose
pain tends to ease with activity. Stiffness is a feature, particularly after
prolonged periods of rest. This is sometimes referred to as 'gelling'. As
pain limits joint movement, secondary muscle atrophy can occur,
leading to weakness. The firm swellings at the PIP joints are Bouchard's
nodes, and the single DIP swelling is a Heberden's node. The age, sex
and obesity are all risk factors for osteoarthritis. Other risk factors include
repetitive occupational trauma, hypermobility, cigarette smoking and a
family history of OA.

Case 18
1 A Osteonecrosis
Osteonecrosis is cellular death of components of bone secondary to an
impaired blood supply. Common sites include the femoral and humeral
heads, femoral condyles and the small bones of the hand and foot. These
are sites with a limited collateral circulation.
Impaired blood flow may be secondary to mechanical alterations (such
as fracture or dislocation), hypercoagulable disorders, embolic
occlusion, vessel wall abnormalities, or pressure on the vessel wall. The
commonest causes are idiopathic, systemic corticosteroid use and
alcoholism.
In the early stages of disease, X-ray changes will be absent. The first
changes may be diffuse osteopenia or a central area of radiolucency with
a sclerotic border. The 'crescent' sign is a subchondral lucency
indicating a subchondral pathological fracture. This is highly suggestive
of osteonecrosis, l ater changes include collapse of the femoral head with
secondary changes of osteoarthritis.

Case 19
1 A Carpal tunnel syndrome
Entrapment neuropathies occur in nearly 50% of patients with
rheumatoid arthritis throughout their lifetime. Carpal tunnel syndrome is
the commonest. The median nerve becomes compressed within the
carpal tunnel, producing characteristic nocturnal dysaesthesias and
sometimes progressing to sensory and motor loss. The median nerve
supplies sensation to the palmar surface of the thumb, index and middle
fingers and the radial side of the ring finger. The thenar motor branch

1 63
RHEUMATOLOGY ANSWERS

supplies index and middle lumbricals, opponens pollicis, abductor


pollicis brevis and flexor pollicis brevis (LOAF).
Other secondary causes of carpal tunnel syndrome include:
C Crystals (gout, pseudogout)
R Rheumatoid arthritis
A Acromegaly, Amyloidosis
M Myxoedema
P Pregnancy
D Diabetes mellitus
A C6 radiculopathy can cause similar sensory changes, but would not
cause motor problems in the hand. However, it is important to be aware
that a 'double crush' can occur and release of a true carpal tunnel
syndrome may not lead to any symptom relief if there is also a
radiculopathy.
Motor neurone disease would not cause any sensory alteration.
The muscle wasting is due to active KA and consequent disuse atrophy.

Case 20
1 D Complex regional pain syndrome (CRPS) type I
Complex regional pain syndrome type I or reflex sympathetic dystrophy
(RSD), is a collection of signs and symptoms affecting an extremity, more
commonly the upper limb. It often follows a noxious, triggering event,
Both limbs can sometimes be affected and, rarely, the face or trunk. If
there is acute atrophy of the bones it is known as Sudeck's atrophy.
The clinical features of RSD are:
• Pain and related sensory abnormalities
• Vasomotor changes
• Sudomotor/oedema
• Motor/trophic.
The pain is severe and often of a burning nature. The affected limb is
often tender with allodynia (pain resulting from a stimulus that would not
normally be painful) and/or hyperalgesia (an exaggerated response to a
painful stimulus). The vasomotor changes can alter with time, often
being warm and red initially, becoming cool and mottled later, then cold
and cyanotic. Sudomotor changes refer to increased sweating.
Motor/trophic changes include weakness, muscle spasms, contractures,
skin atrophy, hypertrichosis, shiny hairless skin and nail changes.
Whilst CRPS type I is often difficult to diagnose, the combination of the
preceding history of trauma and immobilisation, followed by the typical
changes, make this the most likely diagnosis.

164
RHEUMATOLOGY - ANSWERS

Case 21
1 D Amyloidosis
All of the answers would partly explain the findings in this case, but there
is only one unifying diagnosis.
Amyloidosis is a disease of extracellular deposition of insoluble protein.
It can be classified into the following types:
• Systemic
• Hereditary
• CNS
• Ocular
• Localised.
The systemic amyloidoses can be further subclassified into:
• A amyloidosis (chronic inflammation)
• Transthyretin
• Light chain (plasma cell disorders)
• Heavy chain and (ÿ-microglobulin (dialysis-related) amyloidosis.
A amyloidosis (AA) is the most common form of systemic amyloidosis
worldwide. It occurs in response to chronic inflammation, be it infective
or non-infective. Infections precipitating amyloidosis can include TB,
leprosy and bronchiectasis; certain neoplasms can be associated, such as
Hodgkin's disease, non-Hodgkin's lymphoma, renal cell carcinoma and
melanoma. Of the rheumatic diseases, the three most frequently
associated with amyloid are RA, juvenile idiopathic arthritis (JIA) and
ankylosing spondylitis.
Sites of organ involvement include the kidney, leading to nephrotic
syndrome; the liver and spleen, causing hepatosplenomegaly; and the
gastrointestinal tract, causing constipation, diarrhoea and gastrointestinal
bleeding.

Case 22
1 C Systemic lupus erythematosus (SLE)
SLE is a multisystem disorder that can affect almost any organ system. As
such, it can sometimes be very difficult to diagnose. While there are no
strict diagnostic criteria, the American College of Rheumatology (ACR)
have devised classification criteria for the purposes of clinical studies.

The difference between classification and diagnostic criteria means that


an individual may have SLE and not fulfill the classification criteria. As
an example, a patient may have a malar rash, diffuse proliferative

165
RHEUMATOLOGY - ANSWERS

glomerulonephritis and be strongly dsDNA-positive. This palienl has SLE,


ihough does not fulfill the classification criteria. That said, when SEE is
considered as a potential diagnosis, the classification criteria are a very
useful reference point.
The ACR criteria require four or more of the following eleven to be
present:
1 Malar rash - erythema over malar eminences, sparing nasolabial
folds
2 Discoid rash - erythematous raised patches, keratotic scaling and
follicular plugging
3 Photosensitivity
4 Oral ulcers - oral or nasopharyngeal, usually painless
5 Arthritis non-erosive arthritis in two or more joints
6 Serositis - pleuritis, pericarditis
7 Renal disorder- persistent proteinuria or cellular casts
8 Neurological disorder - seizures or psychosis without other cause
9 Haematological disorder:
• haemolytic anaemia with reticulocytosis, or
• leucopcnia <4 x IC/I. on two occasions, or
• lymphopenia <1.5 x I0''/L on two occasions, or
• thrombocytopenia (not drug-induced)
10 Immunological disorder:
• anti dsDNA, or
• anti-Sm, or
• anti-cardiolipin antibodies or lupus anticoagulant
11 Positive ANA.

Anti-nuclear antibodies (ANA) are circulating antibodies that react with


antigens that are located in the cell nucleus. The interior of the cell is an
immune-privileged site, explaining why all individuals do not have
positive ANA. It is detected in the laboratory either by indirect
immunofluorescence assays, defining autoantibodies by their pattern of
binding, or by tests for specific antibodies to individual nuclear antigens
using ELISA, radioimmunoassay or other techniques. These tests identify
extractable nuclear antibodies (ENA). Poorly soluble nucleosomal
antigens, such as dsDNA and histones, are tested using a separate
technique.
A positive ANA test is not diagnostic of a rheumatic, or any other,
disease. Population studies show roughly 1-5% of healthy individuals to
be ANA-positive. The specificity of a positive ANA relates to its titre. The
titre, expressed as 1:40 or 1/40, refers to the 'strength' of the ANA
positivity. It refers to the dilution of serum at which the ANA can still be
detected. A strongly positive ANA is more suggestive of a significant

166
RHEUMATOLOGY ANSWERS

connective tissue disease. Fewer than 1% of healthy controls have a


detectable ANA at litre > 1:320.
While the role of autoantibodies in connective tissue disease
pathophysiology remains unclear, there are clear associations between
particular antibodies and diseases:
• Anti-Ro/SS-A:
primary Sjogren's syndrome (60-80%)
SLE (35%) - associated with photosensitivity, subacute cutaneous
lupus
• Anti-I a/SS-B:
primary Sjogren's syndrome (50%)
SLE (15%)
• Ro + La:
associated with congenital heart block in mothers with SLE
• Anti-UI -RNP:
mixed connective tissue disease
• Anti-Smith (Anti-SM):
highly specific for SLE (5-30%)
• Anti-synthetase, eg Jo-1 :
inflammatory myopathies - polymyositis (30%)
- dermatomyositis (5%)

• Marker for interstitial lung disease in myositis


• Anti-Scl-70:
diffuse cutaneous systemic sclerosis
predictive of interstitial lung disease in DCSS
• dsDNA - SLE, correlation with disease activity, especially lupus
nephritis.
(Percentages refer to proportion of patients with that condition who have
that particular antibody.)

Case 23
1 D Calcium pyrophosphate dihydrate deposition disease
Calcium pyrophosphatedihydrate (CPPD) is a calcium salt that can be
deposited in cartilage-causing chondrocalcinosis on X-rays, as seen in
this case. While this chondrocalcinosis can be asymptomatic, an acute
crystal arthritis can be caused if CPPD crystals are released into a joint.
'Pseudogout' is the term used for this acute arthritis caused by CPPD
crystals. The diagnosis is confirmed by synovial fluid analysis. CPPD
crystals are typically rectangular or rhomboid and are weakly positively
birefringent under polarised light microscopy. This is in contrast to the
needle-shaped crystals of monosodium urate in gout, which are
negatively birefringent.

167
RHEUMATOLOGY - ANSWERS

Basic calcium phosphalc (BCP) crystals are most commonly associated


with an arthropathy at the shoulder, or Milwaukee shoulder. The
destructive arthritis at the finger due to BCP crystals has been called
'Philadelphia finger'. This is a severe degenerative arthritis, often with a
large joint effusion. BCP crystals are composed of calcium
hydroxyapatite and other calcium-containing minerals. Aggregate BCP
crystals can look like 'shiny coins' on light microscopy. They are not
birefringent and therefore cannot be seen on polarised light microscopy.
Cholesterol crystals are most often seen as broad plates with a notched
corner. They are found in synovial fluid, most commonly as a
complication of rheumatoid arthritis.
Cholesterol emboli can occur after an invasive procedure such as
coronary angiography. Clinically this often resembles vasculitis, with a
purpuric rash in the lower extremities.

168
The numbers in this index refer to the chapter and question numbers. The topics shown
may not always be in the question, but may appear in the explanatory answers.

absences, 2.13 back pain


adhesive capsulitis, 4.1 6 ankylosing spondylitis, 4.14
alcoholism, neurological sequelae, 2.8 sudden, 4.5
amyloidosis, 4.21 Behcet's syndrome, 4.10
anaemia bleeding time prolonged, 1.20
autoimmune haemolytic, 1.18 bone marrow
dyserythropoiesis, 1 .8 fibrosis, 1 .8
haemolytic, 1.1, 1.2, 1.6 leukaemic blasts in, 1.10
immune haemolytic, 1.17 bone marrow transplant, 1.5, 1.13
macrocytic, 1.16, 1.18 bone mineral density, 4.5, 4.6
microcytic hypochromic, 1.3 bony erosions
normocytic, 1.10 gout, 4.7
recurrent, 1.20 psoriatic arthritis, 4.2
red cell aplasia, 1.7 botulism, wound infection, 2.1
ANCA (antibody to neutrophil brachial plexus, neuralgic amyotrophy,
cytoplasmic antigens) test, 4.1 2.10
angina, anaemia and, 1.18 bruising
ankylosing hyperostosis, 4.16 acute myeloid leukaemia, 1.10
ankylosing spondylitis, 3.3, 4.14 anaemia, 1 .3
anterior uveitis, 3.2, 3.3 post CML therapy, 1.13
antibiotic, secondary anaemia, 1.17 in pregnancy, 1.15
anticoagulation therapy, 2.3, 2.4
antinuclear antibodies (ANA), 4.22 calcium pyrophosphate dihydrate
antiphospholipid syndrome (APS), 1.14 deposition disease (CPPD), 4.23
antiplatelet therapy, 2.2 cardiac arrhythmia, vitamin B, ,
arthritis deficiency and, 1.16
enteropathic, 4.3 carotid artery dissection, 2.4
in gonococcal infection, 4.10 carpal tunnel syndrome, 4,19
psoriatic, 4.2 cataplexy, 2.11
septic, 4.8 cavernous sinus thrombosis, 2.20
see a/so osteoarthritis (OA); rheumatoid central pontine myelinosis, 2.8
arthritis (RA) cerebellar syndrome, progressive, 2.9
ataxia, 2.1 cheiroarthropathy, diabetic, 4.16
autoimmune haemolysis, 1 .6 chest pain, post-transplant, 1 .5

169
INDEX

chondrocalcinosis, 4.23 glaucoma


coeliac disease, 4.3 acute angle-closure, 3.2
collapse, narcolepsy, 2.1 I secondary, 3.4
complex regional pain syndrome (CRPS) Goldrnann visual field test, 3.1
type 1, 4.20 gonococcal infection, 4.1 0
contusion, meningoencephalitis, 2.1 7 gout, 4.7, 4.8
conjunctivitis, see eyes, redness G6PD deficiency, 1.1, 1.11
cranial nerve palsy, 2.18 granulomas, non-caseating, 1.9
cryoglobulinaemia, 2.15 Guillain llarre syndrome, 2.1
cutaneous systemic sclerosis, 4.4
cytogenetic abnormality, in CML, 1.4 haemolysis, autoimmune, 1.0
hallucinations, 2.13
hands
diabetes mellitus carpal tunnel syndrome, 4.1 9
rheumatic conditions, 4.1 6 psoriatic arthritis, 4.2
vision loss and, 3.3 Raynaud's phenomenon, 4.4
diarrhoea, HTLV-1 and, 1.12 heart murmur, jaundice and, 1 .2
diplopia, 1.7 hepatitis C, cryoglobulinaemia and, 2.15
disseminated gonococcal infection (DGI), hepatosplenomegaly, 1.6
4, 10 hereditary spherocytosis (I IS), 1 . 1
driving, seizures and, 2.13 homonymous hemianopia, 3. 1
drug abuse I lorner's syndrome, 2.4
infective endocarditis and, 2. 1 4 hospital-acquired infections, 1 .5
wound botulism, 2.1 human T-cell leukaemia virus infection
drug-induced myopathy, 2.8 (HTLV-1), 1.12
dyspnoea, lymphadenopathy and, 1 .7 hydrocephalus, 2.5
hyperbilirubinaemia, 1.1
enteropathic arthritis, 4.3 hypercalcaemia, 1.12
enthesitis, 4.10, 4.15 hyperostosis,idiopathic skeletal, 4.1 6
entrapment neuropathy, 4. 1 9 hyperparathyroidism, 4.3
epilepsy, temporal lobe, 2.13 hyponalraemia, alcoholism and, 2.8
exfoliative psoriasis, 4.7 hypothyroidism, 2.9, 4. 12
eye movement disorder, 2.1
eyes infection, reactive arthritis and, 4.1 5
anterior uveitis, 3.2, 3.3 infective endocarditis, spinal abscess and,
diplopia and ptosis, 2.18 2.14
in I lorner's syndrome, 2.4 iron deficiency anaemia, 1 .3
hypertensive retinopathy, 3.4
intraocular pressure, 3.3, 3.4 jaundice
nystagmus, 2.9 heart murmur and, 1.2
opthalmoplegia, 2.20 hepatosplenomegaly and, 1 .0
redness, 3.2, 4.14 pregnancy, 1.15
see also glaucoma; opthalmoplegia; splenomegaly and, 1.1
retina; vision loss joint pain, coeliac disease, 4.3

fibromyalgia, 4.13 knee, hot red with effusion, 4.8


Forestier's disease, 4.1 6
Friederich's ataxia, 2.9 Lambert-Eaton myasthenic syndrome,
frozen shoulder, 4.1 6 2.12
fungal infection, post-transplant, 1.5 lateral medullary syndrome, 2.4
Leishmnnia, tropical splenomegaly, 1 .8
gallstones, pigmented, 1.1, 1.19 leucoerythroblastic blood, 1 .8
Gilbert's syndrome, 1.19 leukaemia

170
INDEX

acute myeloid, 1.10 parvovirus infection, 1.7, 1.19


adult T-cell leukaemia/lymphoma plasma exchange, 1.15
(ATLL), 1.12 poikilocytes, tear-drop, 1.8
chronic lymphocytic, 1.18 polvarthropathy, sarcoidosis, 1.9
chronic myeloid, 1.4, 1.13 polycythaemia, secondary, 1.9
hairy cell, 1.10 polymyalgia rheumatica (PMR), 2.6, 4.12,
human T-cell leukaemia virus infection 4.13
(HTLV-1), 1.12 Pott's disease, 4.11
Listeria monocytogenes infection, 2.1 7 pregnancy
lung carcinoma, 2.9, 2.12 jaundice, 1.15
lung cavitation, 4.1 thrombocytopenia, 1.14
lupus anticoagulant, in pregnancy, 1.14 priapism, splenomegaly and, 1.4
lymphadenopathy, 1.7, 1.9, 1.12 pseudogout, 4.23
lymphoid blast crisis, 1.13 psoriasis, exfoliative, 4.7
lymphoma, 1 .9, 1 .12 psoriatic arthritis, 4.2
lymphoproliferative disorder, 1.6
rash
macrocytosis, hydroxycarbamide therapy, lymphadenopathy and granulomas, 1.9
1.11 rheumatic diseases and, 4.1 0
malaria, 1.8, 1.10, 1.11 in SLE, 4.22
meningitis, 2.5, 2.17 systemic vasculitis, 2.15
meningoencephalitis, 2.1 7 Raynaud's phenomenon, 4.4
mononeuritis multiplex, 2.1 5, 4.1 reactive arthritis, 4 .10, 4. 15
multiple systems atrophy (MSA), 2.16 rectal bleeding, amyloidosis and, 4.21
muscular weakness red cell aplasia, parvovirus-induced, 1.19
in OA, 4.17 red cell membrane disorder, 1.1
proximal, 2.6, 4.12 red cells, anisocytosis and poikilocytosis,
wound botulism, 2.1 1.8
myasthenia gravis, 1.7, 2.19 red eye see eyes, redness
differential diagnosis, 2.1, 2.6, 2.12 reflex sympathetic dystrophy (RSD), 4.20
myelodysplasia, 1.10, 1.16 Reiler's syndrome, 4.1 4
myelofibrosis, 1.8 respiratory tract, in Wegener's
granulomatosis, 4.1
narcolepsy, 2.1 1 retina, cotton wool spots, 3.3, 3.4
neuralgic amyotrophy, 2.10 retinopathy, hypertensive, 3.4
neurocyslicercosis, 2.7 rheumatoid arthritis (RA), 4.9, 4.1 3
neuropathy, asymmetric, 2.15 amyloidosis, 4.21
neutropenia, post-transplant, 1.5 mononeuritis multiplex, 4.1
non-caseating granulomas, 1.9 seropositive, 4.8
steroid therapy, 4.5
occipital cortex lesion, 3.1 T-lymphoproliferative disorder and, 1 .6
oplhalmoplegia, 2.8, 2.20
osteoarthritis (OA), 4.17 sagittal sinus thrombosis, 2.3
osteomalacia, 2.6, 4.13 sarcoidosis, 1 .9
osteonecrosis, 4.18 seizures
osteoporosis, 4.5, 4.6 bacterial meningitis, 2.1 7
neurocysticercosis, 2.7
pancytopenia, 1.10, 1.13 temporal lobe epilepsy, 2.1 3
paraneoplastic cerebellar degeneration, tonic-clonic, 2.8
2.9 septic arthritis, 4.8
Parkinson's disease, 2. 1 6 sickle cell disease, 1.2, 1.11
paroxysmal nocturnal haemoglobinuria sleepiness, cataplexy and, 2.11
(PNH), 1.1 spherocytosis, 1.19

171
IN15 FX

spinal abscess, 2.14 thymoma-associaled myasthenia gravis,


spinal tuberculosis, 4.1 1 1.7
splenomegaly, 1.1, 1.8 transient ischaemic attack, 2.2
stiffness, morning, 4.9, 4.13 tropical splenomegaly, 1.8
stroke tuberculosis, spinal, 4.1 1
occipital cortex lesion, 3.1
prevention, 2.2 ulcerative colitis, thrombosis and, 2.3
sickle cell disease and, 1.2
Strongyloides, T-cell immunity defect vasculitis, Wegener's granulomatosis, 4.1
and, 1.12 ventilator dependence, 2.19
subarachnoid haemorrhage, 2.5 vision loss
systemic lupus erythematosus (SLE), 4.22 gradual, 3.3
sudden, 3.4
T-cell immunity defect, infection and, see also homonymous hemaniopia
1.12 vitamin 13,, deficiency, 1.1 f>
Taenia solium infection, 2.7 vomiting, acute angle-closure glaucoma,
temporal lobe epilepsy, 2.1 3 3.2
tenosynovitis, in DGI, 4.10 von Willebrand's disease, 1 .20
Tensilon0" test, 1.7,2.19
thalassaemia trait, 1 .3 weakness, faliguable proximal, 2.12
thrombocytopenia Weber's syndrome, 2.18
CMI. transformation, 1 .13 Wegener's granulomatosis, 4. 1
pregnancy and, 1.14 Whipple's disease, 4.3
thrombotic, 1.15 white cell count (WCC), splenomegaly
thrombotic thrombocytopenic purpura and, 1 .4
(TTP), 1.15 wound botulism, 2.1

172

You might also like